[obm-l] Re: [obm-l] Re: [obm-l] Re: [obm-l] Soma de raízes cúbicas de cossenos

2020-01-24 Por tôpico saulo nilson
NAO PRECISAVA ENCONTRAR COS5, COS 30=COS3*10, DAÍ ENCONTRA O COS10, DEPOIS
É SÓ SUBSTITUIR.

On Fri, Jan 24, 2020 at 10:23 AM Vanderlei Nemitz 
wrote:

> Como?
>
> Não entendi a ideia...
>
>
> Em sex, 24 de jan de 2020 02:37, saulo nilson 
> escreveu:
>
>> COS 15=COS 30/2
>> COS 15=COS(3*5)
>> DAÍ ENCONTRA O VALOR DE COS5 =COS10/2
>> DAÍ ENCONTRA O VALOR DE COS 10
>>
>> S= F(COS 10) QUE ENCONTRA O VALOR
>>
>> On Sun, Jan 19, 2020 at 8:41 AM Vanderlei Nemitz 
>> wrote:
>>
>>> Bom dia, pessoal!
>>>
>>> Pensei em resolver a seguinte questão associando cos 40°, cos 80° e cos
>>> 160° às raízes da equação cos(3x) = -1/2 e utilizando o arco triplo,
>>> recaindo em uma equação de grau 3. Porém, fica difícil determinar o produto
>>> de 2 em 2 das raízes cúbicas. Alguém conhece uma solução melhor?
>>> Muito obrigado!
>>>
>>> S = (cos 40°)^(1/3) + (cos 80°)^(1/3) +  (cos 160°)^(1/3)
>>>
>>> (Soma das raízes cúbicas de cos 40°, cos 80° e cos 160°)
>>>
>>> --
>>> Esta mensagem foi verificada pelo sistema de antivírus e
>>> acredita-se estar livre de perigo.
>>
>>
>> --
>> Esta mensagem foi verificada pelo sistema de antivírus e
>> acredita-se estar livre de perigo.
>
>
> --
> Esta mensagem foi verificada pelo sistema de antivírus e
> acredita-se estar livre de perigo.

-- 
Esta mensagem foi verificada pelo sistema de antiv�rus e
 acredita-se estar livre de perigo.



[obm-l] Re: [obm-l] Soma de raízes cúbicas de cossenos

2020-01-23 Por tôpico saulo nilson
COS 15=COS 30/2
COS 15=COS(3*5)
DAÍ ENCONTRA O VALOR DE COS5 =COS10/2
DAÍ ENCONTRA O VALOR DE COS 10

S= F(COS 10) QUE ENCONTRA O VALOR

On Sun, Jan 19, 2020 at 8:41 AM Vanderlei Nemitz 
wrote:

> Bom dia, pessoal!
>
> Pensei em resolver a seguinte questão associando cos 40°, cos 80° e cos
> 160° às raízes da equação cos(3x) = -1/2 e utilizando o arco triplo,
> recaindo em uma equação de grau 3. Porém, fica difícil determinar o produto
> de 2 em 2 das raízes cúbicas. Alguém conhece uma solução melhor?
> Muito obrigado!
>
> S = (cos 40°)^(1/3) + (cos 80°)^(1/3) +  (cos 160°)^(1/3)
>
> (Soma das raízes cúbicas de cos 40°, cos 80° e cos 160°)
>
> --
> Esta mensagem foi verificada pelo sistema de antivírus e
> acredita-se estar livre de perigo.

-- 
Esta mensagem foi verificada pelo sistema de antiv�rus e
 acredita-se estar livre de perigo.



[obm-l] Re: [obm-l] Estratégia mais justa

2019-03-02 Por tôpico saulo nilson
mede a área dos quartos e faz ponderação com elas.

On Monday, February 25, 2019, João Maldonado 
wrote:

> Galera, estou tentando dividir um apartamento para 4 pessoas. O preço
> total com IPTU é 3300 reais. Todos os quartos são diferentes e uns são
> melhores que outros subjetivamente. Queria saber qual a melhor estratégia
> de “leilão” para dividir os custos de cada quarto de modo que cada um pague
> o preço justo em cada quarto. O problema é que nem preço justo eu consigo
> definir. Tenho certeza que deve existir uma teoria por trás desse assunto.
> Queria que algumas pessoas me dessem algumas abordagens possíveis do melhor
> modo de definirmos esses preços. Somos todos engenheiros e já estamos há 25
> dias sem definir um preço rsrsrsrs.
>
> Grande abraço!
>
> João M.
> --
> Esta mensagem foi verificada pelo sistema de antivírus e
> acredita-se estar livre de perigo.
>

-- 
Esta mensagem foi verificada pelo sistema de antiv�rus e
 acredita-se estar livre de perigo.



[obm-l] Re: [obm-l] Re: Polinômios

2017-01-14 Por tôpico saulo nilson
termo independente==soma [2n 2k][-1]^2k

2017-01-11 3:31 GMT-02:00 Israel Meireles Chrisostomo <
israelmchrisost...@gmail.com>:

> Para n par e n ímpar
>
> Em 11 de janeiro de 2017 03:29, Israel Meireles Chrisostomo <
> israelmchrisost...@gmail.com> escreveu:
>
>> [image: Imagem inline 1]
>> Qual é o coeficiente líder desse polinômio e o termo independente de
>> x?Alguém poderia me ajudar desenvolvendo o polinômio?
>>
>
>
> --
> Esta mensagem foi verificada pelo sistema de antivírus e
> acredita-se estar livre de perigo.
>

-- 
Esta mensagem foi verificada pelo sistema de antiv�rus e
 acredita-se estar livre de perigo.



Re: [obm-l] Decrescimento

2017-01-14 Por tôpico saulo nilson
crescente
f= x[e^x+e^-x]/2, produto de 2 funções crescentes.

2017-01-12 7:19 GMT-02:00 Bernardo Freitas Paulo da Costa <
bernardo...@gmail.com>:

> 2017-01-11 23:38 GMT-02:00 Israel Meireles Chrisostomo
> :
> >
> > Olá pessoal gostaria de saber se a função é decrescente
>
> Não, pois em infinitos pontos ela tende a infinito.
> --
> Bernardo Freitas Paulo da Costa
>
> --
> Esta mensagem foi verificada pelo sistema de antivírus e
>  acredita-se estar livre de perigo.
>
>
> =
> Instruções para entrar na lista, sair da lista e usar a lista em
> http://www.mat.puc-rio.br/~obmlistas/obm-l.html
> =
>

-- 
Esta mensagem foi verificada pelo sistema de antiv�rus e
 acredita-se estar livre de perigo.



[obm-l] Re: [obm-l] [obm-l] Polinômios

2016-09-06 Por tôpico saulo nilson
p(1\2)=4
(1\4-1\2)4=R=-1

2016-08-02 18:29 GMT-03:00 Daniel Rocha :

> Alguém poderia, por favor, solucionar o problema abaixo:
>
> O resto da divisão de um polinômio P(x) por (2x - 1) é 4; deste modo, o
> resto da divisão de (x^2 - x)*P(x) por (2x - 1) é:
>
> a) -2
> b) -1/2
> c) 1/2
> d) 2
> e) 4
>
> --
> Esta mensagem foi verificada pelo sistema de antivírus e
> acredita-se estar livre de perigo.

-- 
Esta mensagem foi verificada pelo sistema de antiv�rus e
 acredita-se estar livre de perigo.



Re: [obm-l] letras do indice

2016-05-30 Por tôpico saulo nilson
i e j são usados para medir contagens em somatórios, talvez seja por isso.

2016-05-27 16:26 GMT-03:00 Mauricio de Araujo 
:

> i por causa da palavra index? j por causa da proximidade com o i? eu não
> sei...
>
> Em 27 de maio de 2016 14:59,  escreveu:
>
>> Meus amigos um aluno me perguntou pq usamos i j para índice.
>>
>> Alguém sabe a razão? Abraços
>>
>> Hermann
>>
>> --
>> Esta mensagem foi verificada pelo sistema de antivírus e
>> acredita-se estar livre de perigo.
>>
>
>
>
> --
>
> Abraços,
> oɾnɐɹɐ ǝp oıɔıɹnɐɯ
>
>
> --
> Esta mensagem foi verificada pelo sistema de antivírus e
> acredita-se estar livre de perigo.
>

-- 
Esta mensagem foi verificada pelo sistema de antiv�rus e
 acredita-se estar livre de perigo.



[obm-l] Re: [obm-l] duas séries e um resultado

2016-01-25 Por tôpico saulo nilson
a = \sum_{k=0}^\infty \frac{1}{\sqrt{2k+1}} e

b = \sum_{k=1}^\infty \frac{1}{\sqrt{2k}}.

Mostre que a / b = \sqrt{2} - 1.
a\b=soma (1\rq(xk+1)\soma(1\rqxk)
usando Lópital
a\b=soma (-1\2)k(xk+1)^-3\2\(-1\2)k(xk)^-3\2=soma ln(xk+1)\somalnxk=soma
ln(2k+1)\soma(ln2k)=ln(x(x+2)(x+4))\ln(x+1)(x+3)(x+5)=lim
1\(n+1)\1\n  (n-->00)=1
talvez precise de um refinamento com este método.

2016-01-11 12:31 GMT-02:00 Luís :

> Sauda,c~oes,
>
> Um bom 2016 para todos.
>
> Recebi o seguinte problema.
>
> a = \sum_{k=0}^\infty \frac{1}{\sqrt{2k+1}} e
>
> b = \sum_{k=1}^\infty \frac{1}{\sqrt{2k}}.
>
> Mostre que a / b = \sqrt{2} - 1.
>
> Abs,
> Luís
>
>


Re: [obm-l] ajuda(logaritmo)

2016-01-25 Por tôpico saulo nilson
n<0 ,logo n<1\(2-a)

2015-11-10 13:09 GMT-02:00 marcone augusto araújo borges <
marconeborge...@hotmail.com>:

> Seja n um número natural > 1 e seja a um número
> real positivo < 2. Se n = log(1/(2-a)) na base a. Podemos
> afirmar que n < 1/(2-a)?
>
> --
> Esta mensagem foi verificada pelo sistema de antivírus e
> acredita-se estar livre de perigo.
>


Re: [obm-l] Ajuda

2015-10-14 Por tôpico saulo nilson
so resolver a cubica para a e substituir na equação de 2o grau.

2015-10-14 7:57 GMT-03:00 marcone augusto araújo borges <
marconeborge...@hotmail.com>:

> Seja a um número real tal que a^3 = 6(a+1).Mostre que a equação
> x^2 + ax+ a^2 - 6 = 0 não tem raízes reais.
>
> --
> Esta mensagem foi verificada pelo sistema de antivírus e
> acredita-se estar livre de perigo.
>

-- 
Esta mensagem foi verificada pelo sistema de antiv�rus e
 acredita-se estar livre de perigo.



[obm-l] Re: [obm-l] Problema muito bacana de teoria dos números

2015-08-06 Por tôpico saulo nilson
d4-1=11
d4=12
d1=1
d2=2
d3=
d11=(1+2+12)d8=15*17=255
1,2,3,12,13,14,15,17,18,19,255, produto deles.

2015-08-06 13:14 GMT-03:00 Mauricio de Araujo mauricio.de.ara...@gmail.com
:

 Um número natural N tem exatamente 12 divisores (incluindo 1 e N), tais
 que, colocados em ordem crescente temos d1  d2  d3  ...  d12.
 Sabe-se que o divisor que possui o índice d4 - 1 é igual ao produto (d1 +
 d2 + d4).d8. Achar N.

 --
 Abraços

 oɾnɐɹɐ ǝp oıɔıɹnɐɯ


 --
 Esta mensagem foi verificada pelo sistema de antivírus e
 acredita-se estar livre de perigo.

-- 
Esta mensagem foi verificada pelo sistema de antiv�rus e
 acredita-se estar livre de perigo.



[obm-l] Re: [obm-l] Problema muito bacana de teoria dos números

2015-08-06 Por tôpico saulo nilson
d4-1=11
d4=12
d1=1
d2=2
d3=
d11=(1+2+12)d8=15*17=255
1,2,3,12,13,14,15,17,18,19,255,256

2015-08-06 13:14 GMT-03:00 Mauricio de Araujo mauricio.de.ara...@gmail.com
:

 Um número natural N tem exatamente 12 divisores (incluindo 1 e N), tais
 que, colocados em ordem crescente temos d1  d2  d3  ...  d12.
 Sabe-se que o divisor que possui o índice d4 - 1 é igual ao produto (d1 +
 d2 + d4).d8. Achar N.

 --
 Abraços

 oɾnɐɹɐ ǝp oıɔıɹnɐɯ


 --
 Esta mensagem foi verificada pelo sistema de antivírus e
 acredita-se estar livre de perigo.

-- 
Esta mensagem foi verificada pelo sistema de antiv�rus e
 acredita-se estar livre de perigo.



[obm-l] Re: [obm-l] Polinômios

2015-06-18 Por tôpico saulo nilson
a+b+c=17
ab+ac+bc=m
abc=n^2
abc tem que dar um quadrado perfeito
a=6,b=3,c=8
n=12
m=92

2015-05-18 7:23 GMT-03:00 marcone augusto araújo borges 
marconeborge...@hotmail.com:

 Encontrar todos os inteiros positivos m e n tais que todas as soluções de
 x^3 - 17x^2 + mx - n^2 = 0 são inteiras

 --
 Esta mensagem foi verificada pelo sistema de antivírus e
 acredita-se estar livre de perigo.


-- 
Esta mensagem foi verificada pelo sistema de antiv�rus e
 acredita-se estar livre de perigo.



[obm-l] Re: [obm-l] Polinômios

2015-06-18 Por tôpico saulo nilson
e uma soluçao


2015-06-18 14:13 GMT-03:00 saulo nilson saulo.nil...@gmail.com:

 a+b+c=17
 ab+ac+bc=m
 abc=n^2
 abc tem que dar um quadrado perfeito
 a=6,b=3,c=8
 n=12
 m=92

 2015-05-18 7:23 GMT-03:00 marcone augusto araújo borges 
 marconeborge...@hotmail.com:

 Encontrar todos os inteiros positivos m e n tais que todas as soluções de
 x^3 - 17x^2 + mx - n^2 = 0 são inteiras

 --
 Esta mensagem foi verificada pelo sistema de antivírus e
 acredita-se estar livre de perigo.




-- 
Esta mensagem foi verificada pelo sistema de antiv�rus e
 acredita-se estar livre de perigo.



[obm-l] Re: [obm-l] Polinômios

2015-06-18 Por tôpico saulo nilson
a+b+c=17
ab+ac+bc=m
abc=n^2
abc tem que dar um quadrado perfeito
a=6,b=3,c=8
n=12
m=90

2015-06-18 14:13 GMT-03:00 saulo nilson saulo.nil...@gmail.com:

 e uma soluçao


 2015-06-18 14:13 GMT-03:00 saulo nilson saulo.nil...@gmail.com:

 a+b+c=17
 ab+ac+bc=m
 abc=n^2
 abc tem que dar um quadrado perfeito
 a=6,b=3,c=8
 n=12
 m=92

 2015-05-18 7:23 GMT-03:00 marcone augusto araújo borges 
 marconeborge...@hotmail.com:

 Encontrar todos os inteiros positivos m e n tais que todas as soluções
 de x^3 - 17x^2 + mx - n^2 = 0 são inteiras

 --
 Esta mensagem foi verificada pelo sistema de antivírus e
 acredita-se estar livre de perigo.





-- 
Esta mensagem foi verificada pelo sistema de antiv�rus e
 acredita-se estar livre de perigo.



Re: [obm-l] RES: soma finita??? corrigindo

2015-06-09 Por tôpico saulo nilson
S=d/dx soma  x^n para x=2

2015-06-02 10:44 GMT-03:00 Ralph Teixeira ralp...@gmail.com:

 Suponho que seja 2^(n-1)*n?

 Seja
 1S = 1.1+2.2+4.3+8.4+...+2^(n-1).n
 Entao, botando um 0 na frente para alinhar do jeito que eu quero:
 2S = 0.0+2.1+4.2+8.3+...+2^(n-1).(n-1)+2^n.n
 Subtraindo e vendo a PG negativa:
 S =  -1 -2 -4 -8... -2^(n-1) + 2^n.n = 2^n.n - 2^n + 1= 2^n.(n-1) + 1
 Divida por n, e acabou!

 Abraco, Ralph.

 2015-06-01 22:38 GMT-03:00 Vitório Batista Lima da Silva 
 vitorio.si...@trf1.jus.br:


 
 De: owner-ob...@mat.puc-rio.br [owner-ob...@mat.puc-rio.br] em Nome de
 Vitório Batista Lima da Silva
 Enviado: segunda-feira, 1 de junho de 2015 19:13
 Para: 'obm-l@mat.puc-rio.br'
 Assunto: [obm-l] soma finita???

 Nobres,

 Como procedo:

 Calcule a média aritmética das seguintes quantidades

 1;4;12;32; ...; (2^n-1)*n

 Vitório Gauss

 --
 Esta mensagem foi verificada pelo sistema de antivírus e
 acredita-se estar livre de perigo.

 --
 Esta mensagem foi verificada pelo sistema de antivírus e
  acredita-se estar livre de perigo.


 =
 Instruções para entrar na lista, sair da lista e usar a lista em
 http://www.mat.puc-rio.br/~obmlistas/obm-l.html
 =



 --
 Esta mensagem foi verificada pelo sistema de antivírus e
 acredita-se estar livre de perigo.


-- 
Esta mensagem foi verificada pelo sistema de antiv�rus e
 acredita-se estar livre de perigo.



Re: [obm-l] Probabilidade Random quadratic equations

2015-03-07 Por tôpico saulo nilson
r^2s
P=lim (n--oo )(n-[sqrts])/n=(n-n/k)/n=1-1/k

2015-03-03 22:57 GMT-03:00 Mauricio de Araujo mauricio.de.ara...@gmail.com
:

 eis o livro:
 https://mega.co.nz/#!O5ElSAyI!LmCHjd1xcLfex6fpH8I7pnGplcejFi4nAQRojHYgBTI

 Em 3 de março de 2015 18:59, Douglas Oliveira de Lima 
 profdouglaso.del...@gmail.com escreveu:

 Acho que encontrei a questão original,  num livro do professor de
 matemática estatística Frederick Mosteller da universidade de Harvard
 publicado em 1965 com o título FIFA CHALLENGING PROBLEMS IN PROBABILITY
 questão número 50 inclusive existem mais problemas fantásticos,  estou
 lendo
 Douglas oliveira
 Em 03/03/2015 13:47, Ralph Teixeira ralp...@gmail.com escreveu:

 Impossivel responder sem que se de uma ideia da distribuicao de
 probabilidade atendidas por r e s

 (Eu sou o chato da lista que reclama que tem muito problema de
 probabilidade que nao tem enunciado preciso...)

 Uma possibilidade eh tomar r e s distribuidos uniformemente e
 independentemente no intervalo [-A,A], e entao tomar A-+Inf.

 Agora sim: para ter raiz real devemos ter s=r^2. No quadrado
 [-A,A]x[-A,A] do plano rs, a regiao ruim (sem raiz real) eh acima da
 parabola, cuja area eh

 Int [r=-raiz(A), r=raiz(A)] (A-r^2) dr =
 2Araiz(A)-2Araiz(A)/3=4Araiz(A)/3

 (ok, usei aqui A1, que eh razoavel jah que vamos tomar A-+Inf)

 Entao a probabilidade ruim seria isto dividido por 4A^2, isto eh,
 numero/raiz(A), que vai para 0 quando A-+Inf.

 Assim, nesse sentido, a resposta eh a probabilidade de ter raiz real eh
 1 (o que NAO significa garantia de ter raiz real).

 Abraco, Ralph

 P.S.: Se esta resposta lhe parece estranha, experimente desenhar a
 parabola y=x^2 no quadrado [-10,10]x[-10,10] e observe a area que fique
 acima dela em comparacao com o quadrado todo. Agora aumente o quadrado para
 [-100,100]x[-100,100], depois 1000, depois 1, e veja o que acontece --
 a regiao acima da parabola fica proporcionalmente cada vez menor, e tende a
 0.

 2015-03-03 10:55 GMT-03:00 Douglas Oliveira de Lima 
 profdouglaso.del...@gmail.com:

 Olá caros amigos, recebi um problema esta semana, e gostaria de uma
 ajuda (se possível), dos senhores.
 Pesquisei esse problema na internet e achei algumas divergências de
 soluções.
 Eis o problema:
 Qual a probabilidade da equação do segundo grau x^2+2rx+s=0 ter raiz
 real?



 Agradeço desde já a ajuda.
 Douglas Oliveira.


 --
 Esta mensagem foi verificada pelo sistema de antivírus e
 acredita-se estar livre de perigo.



 --
 Esta mensagem foi verificada pelo sistema de antivírus e
 acredita-se estar livre de perigo.


 --
 Esta mensagem foi verificada pelo sistema de antivírus e
 acredita-se estar livre de perigo.




 --
 Abraços

 oɾnɐɹɐ ǝp oıɔıɹnɐɯ


 --
 Esta mensagem foi verificada pelo sistema de antivírus e
 acredita-se estar livre de perigo.


-- 
Esta mensagem foi verificada pelo sistema de antiv�rus e
 acredita-se estar livre de perigo.



Re: [obm-l] Desigualdades Numeros Naturais

2015-02-25 Por tôpico saulo nilson
a=c+d-d
2015-02-13 10:06 GMT-02:00 luiz silva luizfelipec...@yahoo.com.br:

  Pessoal,

 Dados dois numeros naturais a, b, c e d onde :

 ac
 db

 b é multiplo de 2 e os outros numeros são impares

 Quais as condições para que tenhamos

 a + b  c + d

 cd  ab

 Abs
 Felipe



 --
 Esta mensagem foi verificada pelo sistema de antivírus e
 acredita-se estar livre de perigo.


-- 
Esta mensagem foi verificada pelo sistema de antiv�rus e
 acredita-se estar livre de perigo.



[obm-l] Re: [obm-l] Re: [obm-l] Re: [obm-l] Primos em Potências - Uma ajuda

2015-02-25 Por tôpico saulo nilson
10^2n-10^n-1=pn
9...9899.99=pn
 =99..099..9+9...000-100000=
=9...999.99-1=9*11..-10^n
nao e primo quando11.e potencia par de algum numero  n
e par
2015-02-03 8:00 GMT-02:00 Bernardo Freitas Paulo da Costa 
bernardo...@gmail.com:

  Em 24 de janeiro de 2015 08:23, Richard Vilhena 
 ragnarok.liv...@gmail.com
  escreveu:
 
  Em que condições 10^2n - 10^n  -1 é um  número primo?
 
  Exemplos: 10^2 -  10- 1 = 89(primo)
   10^4 - 10^2 - 1 = 9899( não é primo)
 
  Obrigado.

 2015-02-03 0:36 GMT-02:00 terence thirteen peterdirich...@gmail.com:
  É bem provável que em poucos valores. Basicamente é saber quando X^2-X-1
 é
  primo, X=10^n.
 
  Mas (X^3+1)/(X+1) não parece ser um bom gerador para tais primos,
 Mas X^3 + 1 = (X+1)(X^2 - X + 1). Tem um 2 sobrando nas suas contas.

 Para n = 30, o PARI acha que só n = 1,6 e 9 servem.

 Abraços,
 --
 Bernardo Freitas Paulo da Costa

 --
 Esta mensagem foi verificada pelo sistema de antivírus e
  acredita-se estar livre de perigo.


 =
 Instru�ões para entrar na lista, sair da lista e usar a lista em
 http://www.mat.puc-rio.br/~obmlistas/obm-l.html
 =


-- 
Esta mensagem foi verificada pelo sistema de antiv�rus e
 acredita-se estar livre de perigo.



[obm-l] Re: [obm-l] Questão interessante

2015-02-22 Por tôpico saulo nilson
x-r+x+x+r=180
x=60
(y-w)a+y(b)+(y+w)c=acrq3/2
b^2=a^2+c^2-ac
sen(60-r)=h1/b


2015-02-21 13:39 GMT-02:00 marcone augusto araújo borges 
marconeborge...@hotmail.com:

  Espero que alguém goste assim como eu gostei:

 As medidas dos ângulos internos de um triângulo estão em PA e as medidas
 das alturas do mesmo triângulo estão em PA.Prove que o triângulo é
 equilátero.

 --
 Esta mensagem foi verificada pelo sistema de antivírus e
 acredita-se estar livre de perigo.


-- 
Esta mensagem foi verificada pelo sistema de antiv�rus e
 acredita-se estar livre de perigo.



[obm-l] Re: [obm-l] Questão interessante

2015-02-22 Por tôpico saulo nilson
x-r+x+x+r=180
x=60
(y-w)a+y(b)+(y+w)c=acrq3/2
b^2=a^2+c^2-ac
sen(60-r)=h2/b=h3/a
sen(60+r)=h1/b=h3/c
h3/h2=a/b
h3/h1=c/b
h1/h2=a/c
(h3-h2)/h2=(a-b)/b
(h2-h1)/h1=(c-a)/a
w/h2=(a-b)/b
w/h1=(c-a)/a
h1/h2=(a-b)a/(c-a)b=a/c
(c-a)b=(a-b)c
cb-ab=ac-bc
2bc=ac+ab
b^2=a^2+c^2-ac
b^2=4b^2c^2/(b+c)^2 +c^2-2bc^2/(b+c)
b^2/c^2 (b+c)^2=4b^2+b^2+c^2 -2b^2=3b^2+c^2
b^2(b^2+2bc+c^2)=c^4+3b^2c^2
b^4+2b^3c=c^4+2b^2c^2
b=c uma das respostas
logo a=b=c triângulo equilátero

-02-22 15:26 GMT-03:00 saulo nilson saulo.nil...@gmail.com:

 x-r+x+x+r=180
 x=60
 (y-w)a+y(b)+(y+w)c=acrq3/2
 b^2=a^2+c^2-ac
 sen(60-r)=h1/b


 2015-02-21 13:39 GMT-02:00 marcone augusto araújo borges 
 marconeborge...@hotmail.com:

  Espero que alguém goste assim como eu gostei:

 As medidas dos ângulos internos de um triângulo estão em PA e as medidas
 das alturas do mesmo triângulo estão em PA.Prove que o triângulo é
 equilátero.

 --
 Esta mensagem foi verificada pelo sistema de antivírus e
 acredita-se estar livre de perigo.




-- 
Esta mensagem foi verificada pelo sistema de antiv�rus e
 acredita-se estar livre de perigo.



Re: [obm-l] Inteiros de novo

2015-01-08 Por tôpico saulo nilson
2^n=(2k+1)(2x+1)^2-1=(2k+1)(4x^2+4x+1)-1=2k(4x^2+4x+1)+4x^2+4x=
2(k(4x^2+4x+1)+2x^2+2x)
2^(n-1)=(4k+2)x^2+(4k+2)x+k
delta=16k^2+16k+4-16k^2-8k=8k+4
x=(-2k-1+-sqrt(2k+1))/2(2k+1)
2^(n)=(2(2k+1)x+2k+1-sqrt(2k+1))(2(2k+1)x+2k+1+sqrt(2k+1))/(2k+1)
2k+1=y^2
y^22^n=(2y^2x+y^2-y)(2y^2x+y^2+y)
2^n=(2yx+y-1)(2yx+y+1)
dois numeros quase consecutivos potencia de 2
2yx+y-1=2
2yx+y+1=4
n=3
2yx+y=3, y(2x+1)=3

2015-01-05 17:32 GMT-02:00 marcone augusto araújo borges 
marconeborge...@hotmail.com:

  Determine todos os inteiros positivos n tais que (2^n +1) / n^2 é
 inteiro

 --
 Esta mensagem foi verificada pelo sistema de antivírus e
 acredita-se estar livre de perigo.


-- 
Esta mensagem foi verificada pelo sistema de antiv�rus e
 acredita-se estar livre de perigo.



Re: [obm-l] Integral interessante

2015-01-07 Por tôpico saulo nilson
x=ae^y
dx=ae^ydy
I=Int (lna+y)e^ydy/a(e^2y+1)=(1/2a)(lnaInt dy/coshy+
+Int ydy/coshy)=
=(1/2a)(-1/2 i (Li_2(-i e^(-y))-Li_2(i e^(-y))-y(log(1-i e^(-y))-log(1+i
e^(-y
y=-oo e oo

ine
2015-01-07 9:23 GMT-02:00 Artur Costa Steiner steinerar...@gmail.com:

 Para a  0, determinar

 I(a) = Int (0, oo) ln(x)/(x^2 + a^2)

 Abraços.

 Artur Costa Steiner
 --
 Esta mensagem foi verificada pelo sistema de antivírus e
  acredita-se estar livre de perigo.


 =
 Instruções para entrar na lista, sair da lista e usar a lista em
 http://www.mat.puc-rio.br/~obmlistas/obm-l.html
 =


-- 
Esta mensagem foi verificada pelo sistema de antiv�rus e
 acredita-se estar livre de perigo.



Re: [obm-l] Prove que...

2015-01-06 Por tôpico saulo nilson
(4a^2-1)^2=K(4ab-1)=k4b(a-1/4b)
a=1/4b  e raiz
4b^2-1=0
b=+-1/2
como b e inteiro so podemos ter
a=b
pois (4a^2-1)^2=0mod(4a^2-1)
2015-01-05 17:48 GMT-02:00 marcone augusto araújo borges 
marconeborge...@hotmail.com:

  Prove que se a e b são dois inteiros positivos tais que 4ab - 1 divide
 (4a^2 - 1)^2
 então a = b

 --
 Esta mensagem foi verificada pelo sistema de antivírus e
 acredita-se estar livre de perigo.


-- 
Esta mensagem foi verificada pelo sistema de antiv�rus e
 acredita-se estar livre de perigo.



Re: [obm-l] Inteiros positivos

2014-12-26 Por tôpico saulo nilson
n2^(n-1)=(m-1)(m+1)
n=2^zw
m-1=2^xk
m+1=2^yu
w2^(n+z-1)=2^(x+y)ku
ku=w
n+z-1=x+y
1=2^(y-1)u-2^(x-1)k
soluçoes
u=29
y=1
k=7
x=3
w=203
n+z=5
2014-12-26 1:16 GMT-02:00 Ralph Teixeira ralp...@gmail.com:

  Ficou subentendido que m e n sao naturais positivos.

 n=1 nao serve, entao o lado direito eh par. Entao m eh impar, digamos,
 m=2k+1. Entao fica n.2^(n-1)=4k(k+1).

 Como n=2 nao serve, podemos escrever n.2^(n-3)=k(k+1). Note que n=4 nao
 serve, e n=5 dah aquela solucao.

 Agora, o problema eh que um dos dois fatores do lado direito eh impar,
 entao tem que dividir o fator n do lado esquerdo. Isto significa que n=k,
 o que diz que o lado esquerdo vai ser muito grande, e a igualdade nao vai
 valer.

 Mais exatamente, prove primeiro por inducao que 2^s  2s para s=3. Entao,
 se n=6, temos k(k+1) = n.2^(n-3)  n.2.(n-3) = 2k(k-3). Daqui vem
 k+12k-6, isto eh, k7. Teste os poucos casos que sobram e acabou.

 Abraco, Ralph.

 P.S.: Ou teste n=6 que nao dah nada; depois mostre que 2^s = 4s para
 s=4; e use agora k(k+1) = n.2^(n-3) = n.4.(n-3) = 4k(k-3). Daqui vem
 k+1=4k-12, isto eh, k=4, e nao ha mais nada para testar.


 2014-12-25 23:03 GMT-02:00 marcone augusto araújo borges 
 marconeborge...@hotmail.com:

  n.2^(n-1) + 1 = m^2.Como resolver?
 n = 5 e m = 9.Outras soluções?

 --
 Esta mensagem foi verificada pelo sistema de antivírus e
 acredita-se estar livre de perigo.



 --
 Esta mensagem foi verificada pelo sistema de antivírus e
 acredita-se estar livre de perigo.


-- 
Esta mensagem foi verificada pelo sistema de antiv�rus e
 acredita-se estar livre de perigo.



[obm-l] Re: [obm-l] Sequência Complicada

2014-12-21 Por tôpico saulo nilson
2^11,3^5,2^12,3^6,2^14,3^6*6,2^14*33,3^6*6*8,2^17*3...
2014-12-19 8:08 GMT-02:00 Richard Vilhena ragnarok.liv...@gmail.com:

 Prezados, não consigo encontrar o termo geral desta sequência onde são
 dados os nove primeiros termos:

 2^3, 3^4 , 2^4 , 3^5 , 2^6, 3^5 × 5, 2^7 × 3, 3^5 × 5 × 7, 2^10 × 3, …

 Agradeço a ajuda.
 [[ ]]'s

 --
 Esta mensagem foi verificada pelo sistema de antivírus e
 acredita-se estar livre de perigo.

-- 
Esta mensagem foi verificada pelo sistema de antiv�rus e
 acredita-se estar livre de perigo.



[obm-l] Re: [obm-l] Seleção num conjunto de inteiros

2014-12-21 Por tôpico saulo nilson
+31+47+-19+11+41-13=108
2014-12-20 9:02 GMT-02:00 Richard Vilhena ragnarok.liv...@gmail.com:

  Caros colegas da lista, solicito uma ajuda nesses dois problemas.

 Problema 1:

  Dado um conjunto de inteiros:

 {-7,11,-13,17,-19,23,-29,31,-37,41,-43,47}

   Selecione alguns elementos distintos desse conjunto (sem repetição) tal
 que a soma deles seja igual a  108.

  Problema 2:

  Dado um conjunto de números inteiros:

 {-101, 103, -107,109, -113,127, -131, 137, -139, 149, -151, 157, -163,
 167, -173, 179, -181, 191, -193, 197}

   Selecione alguns elementos distintos desse conjunto (sem repetição) tal
 que a soma deles seja igual a 1058 .


 Obrigado por qualquer ajuda.

 --
 Esta mensagem foi verificada pelo sistema de antivírus e
 acredita-se estar livre de perigo.

-- 
Esta mensagem foi verificada pelo sistema de antiv�rus e
 acredita-se estar livre de perigo.



Re: [obm-l] Problema interessante de EDO

2014-12-21 Por tôpico saulo nilson
y=A(x)senx
y´=A´senx+Acosx
y=Acosx+A´cosx+A´cosx-Asenx
A+2A´=0
A´=u
u´+2u=0
lnu=-2x+c
u=Ce^(-2x)
A(x)=C1e^(-2x)+C2
y(x)=(C1e^(-2x)+C2)senx=0
x=2npi que corresponde a infinitos zeros
2014-12-19 19:50 GMT-02:00 Artur Costa Steiner steinerar...@gmail.com:

 Seja g uma função contínua em [a, oo) tal que, para todo x neste
 intervalo, tenhamos g(x)  m  0. Mostre que, se y é solução da EDO

 y'' + g(x) y = 0

 então y tem uma infinidade de zeros em [a, oo).

 Abraços.

 Artur Costa Steiner
 --
 Esta mensagem foi verificada pelo sistema de antivírus e
  acredita-se estar livre de perigo.


 =
 Instru�ões para entrar na lista, sair da lista e usar a lista em
 http://www.mat.puc-rio.br/~obmlistas/obm-l.html
 =


-- 
Esta mensagem foi verificada pelo sistema de antiv�rus e
 acredita-se estar livre de perigo.



Re: [obm-l] Problema interessante de EDO

2014-12-21 Por tôpico saulo nilson
y(x)=A(x)senx+B(x)cosx
y(x)=0
sen(x+u)=0
x+u=2npi
x=2npi-u que sao infinitos valores de n para obter x.
2014-12-19 19:50 GMT-02:00 Artur Costa Steiner steinerar...@gmail.com:

 Seja g uma função contínua em [a, oo) tal que, para todo x neste
 intervalo, tenhamos g(x)  m  0. Mostre que, se y é solução da EDO

 y'' + g(x) y = 0

 então y tem uma infinidade de zeros em [a, oo).

 Abraços.

 Artur Costa Steiner
 --
 Esta mensagem foi verificada pelo sistema de antivírus e
  acredita-se estar livre de perigo.


 =
 Instru�ões para entrar na lista, sair da lista e usar a lista em
 http://www.mat.puc-rio.br/~obmlistas/obm-l.html
 =


-- 
Esta mensagem foi verificada pelo sistema de antiv�rus e
 acredita-se estar livre de perigo.



Re: [obm-l] Provar que...

2014-12-20 Por tôpico saulo nilson
100!  50^100

100!=(101-1)(100-1)(99-1);(2-1)=polinmio cujas raizes são  2 a 101.
x^100+(103)*50x^99++2*3**101
50^100=(1-51)^100=C(100,0)x^100*51^0+C(100,1)x^99*51^1
aproximando por serie
ln100!100+50*51=50*532650
ln50^100=100*515100
ln100!ln50^100---100!50^100



2014-12-20 16:58 GMT-02:00 Jeferson Almir jefersonram...@gmail.com:

 Use médias ... M.A  M.G
 Algo assim (1+ 2 + 3+...+100)/100 = (1.2.3 ..100)^1/100
 Do lado esquerdo vc usa soma de gauss ai fica (50.101)/100  (100!)^1/100
  vou ver se faço as conta aqui mais detalhado e mando...


 Em sábado, 20 de dezembro de 2014, Bernardo Freitas Paulo da Costa 
 bernardo...@gmail.com escreveu:

 2014-12-20 0:22 GMT-02:00 Maikel Andril Marcelino 
 maikinho0...@hotmail.com:
  Mas 50x51  50², temos um problema!

 49*52  50*50 também. Talvez seja melhor cancelar o 50 que aparece dos
 dois lados, daí fica 49*51, 48*52, etc, que são (a-b)*(a+b)  a*a. Mas
 daí vai sobrar o 100. Falta pouco.

  From: dr.dhe...@outlook.com
  To: obm-l@mat.puc-rio.br
  Subject: RE: [obm-l] Provar que...
  Date: Sat, 20 Dec 2014 05:14:46 +0300
 
 
  Tenta reagrupar 100!, talvez algo como (1*100)(2*99)(3*98)...(50*51),
 dai
  você terá 50 produtos, cada um deles é equiparável a 50² (a saber
 menor),
  dai tem que argumentar um pouquinho, mas acho que sai.
 
  Abraços
  Edu
 
  
  From: maikinho0...@hotmail.com
  To: obm-l@mat.puc-rio.br
  Subject: [obm-l] Provar que...
  Date: Sat, 20 Dec 2014 04:44:26 +0300
 
  100!  50^100, não estou conseguindo galera. Um abraço Carlos Gomes.

 --
 Bernardo Freitas Paulo da Costa

 --
 Esta mensagem foi verificada pelo sistema de antivírus e
  acredita-se estar livre de perigo.


 =
 Instru�ões para entrar na lista, sair da lista e usar a lista em
 http://www.mat.puc-rio.br/~obmlistas/obm-l.html
 =


 --
 Esta mensagem foi verificada pelo sistema de antivírus e
 acredita-se estar livre de perigo.


-- 
Esta mensagem foi verificada pelo sistema de antiv�rus e
 acredita-se estar livre de perigo.



Re: [obm-l] Derivadas parciais

2014-12-19 Por tôpico saulo nilson
f(x,y)=xy+C na segunda
2014-12-17 20:18 GMT-02:00 Ralph Teixeira ralp...@gmail.com:

 1) Supondo que o dominio eh R^2: se a derivada de algo com relacao a x eh
 zero, entao essa coisa nao depende de x, certo?

 Entao se d2f/dxdy=0, isto significa que df/dy=h(y), onde h(y) eh uma
 funcao qualquer que soh depende de y.

 Agora integre isso: f(x,y)=Int h(y) dy = H(y)+C onde H eh uma
 anti-derivada de h(y) e esse C eh uma constante nao, pera, constante
 *em relacao a y* que eh a variavel de integracao! Entao C pode depender de
 x, isto eh

 f(x,y)=H(y)+C(x), como voce disse.

 2) Essa eh a EDP da onda... Voce pode fazer uma troca de variaveis,
 colocando w=x+y e z=x-y, ou seja, criando a funcao g(w,z)=f(x,y) onde w=x+y
 e z=x-y.

 Agora substitua f(x,y)=g(x+y,x-y) na EDP para encontrar uma nova EDP para
 g(w,z)... (voce estah supondo que f eh C^2 para usar a Regra da Cadeia, mas
 eu imagino que eh isso que voce quer). Vai dar um bom trabalho, e voce vai
 descobrir que...

 ...nah, nao vou estragar a surpresa. :)

 Abraco, Ralph.

 P.S.: Se voce tiver condicoes iniciais do tipo f(x,0)=F(x) e
 df/dy(x,0)=G(x), tem a formula de d'Alembert que resolve isso.



 2014-12-17 17:06 GMT-02:00 João Maldonado joao_maldona...@hotmail.com:

  Fala galera,
 Fiquei um tempo sumido mas voltei para pedir a ajuda de vocês em uma
 questão de cálculo.

 Como resolver as seguintes equações?
 1) d2f/dxdy = 0
 2) d2f/dx2 = d2f/dy2

 Ta meio ruim a formatação, mas é o máximo que consegui por aqui.
 Estou no primeiro ano de engenharia, ainda não aprendi equações
 diferenciais parciais, e isso tava no tópico sobre cálculo 2 (limite,
 derivada e integral em mais de uma variavel). Alguém sabe como posso
 resolver?

 A primeira para mim é meio óbvio que dá a(x)  + b(y), mas não sei fazer
 isso formalmente.

 [] 's
 João

 --
 Esta mensagem foi verificada pelo sistema de antivírus e
 acredita-se estar livre de perigo.


 --
 Esta mensagem foi verificada pelo sistema de antivírus e
 acredita-se estar livre de perigo.


-- 
Esta mensagem foi verificada pelo sistema de antiv�rus e
 acredita-se estar livre de perigo.



Re: [obm-l] prob

2014-12-06 Por tôpico saulo nilson
C15,3 - somaC(i+3-1,3) (i=6 a 9)=C15,3-C11,3-C(10,3)-C(9,3)-C(8,3)
2014-12-06 9:34 GMT-02:00 Silas Gruta silasgr...@gmail.com:

 Olá bom dia mestres,


 poderiam ajudar com a seguinte questão?

  *Em uma urna existem bolas numeradas de 1 a 15. De quantas maneiras
 podemos retirar 3 bolas da urna, sendo que a soma delas não seja menor que
 10?*

 *a) 312*

 *b) 449*

 *c) 455*

 *d) 412*

 *e) 378*

 --
 Silas Gruta

 --
 Esta mensagem foi verificada pelo sistema de antivírus e
 acredita-se estar livre de perigo.

-- 
Esta mensagem foi verificada pelo sistema de antiv�rus e
 acredita-se estar livre de perigo.



[obm-l] Re: [obm-l] Re: [obm-l] Potenciação

2014-11-28 Por tôpico saulo nilson
4 714 714714 fica repetindo na soma dos diigitos.
2014-11-23 22:00 GMT-02:00 Iuri Rezende Souza iuri_...@hotmail.com:

  Olá!

 A primeira congruência:

 Como 31 tem mesmo resto que 4 ao dividir por 9, 31*31*31*...*31 (n vezes)
 tem o mesmo resto que 4*4*4*...*4 (n vezes) ao dividir por 9. Logo, 31^31 =
 4^31 (mod 9)

 A segunda congruência:

 Observe o que acontece com os restos (mod 9) ao multiplicar o 4 várias
 vezes. Temos a sequência 4, 7, 1, 4, 7, 1, 4, 7, 1, 4, 7, 1, ..., que é
 periódica com período 3. Então basta olhar o resto do expoente (31) por 3.

 Outro modo de ver isso é qual potência de 4 tem resto 1 ao ser dividida
 por 9 (isso é possível, já que 4 e 9 são primos entre si). 4^3 é essa
 potência. Então podemos separar os termos do produto de 3 em 3. Observe que
 4^31 = 4*4*4*4*4*...*4 = (4*4*4)*(4*4*4)*(4*4*4)*(4*4*4)*(4*4*4)*(4*4*4)*4
 = ((4^3)^6)*4. Sabendo que 4^3 tem resto 1 ao ser dividido por 9, o resto
 desse número é igual a (1^6)*4 = 4.

 Mudando um pouco de problema, um exemplo disso em que podemos simplificar
 uma potência com aritmética modular é o critério da divisão por 9 na base
 decimal. O número com algarismos abcd é igual a a*10^3 + b*10^2 + c*10^1 +
 d*10^0. Observe que 10 deixa mesmo resto que 1 ao ser dividido por 9, ou,
 em outras palavras, 10 = 1 (mod 9). Assim, a*10^3 + b*10^2 + c*10^1 +
 d*10^0 = a*1^3 + b*1^2 + c*1^1 + d*1^0 (mod 9). Continuando, a*1^3 + b*1^2
 + c*1^1 + d*1^0 = a+b+c+d (mod 9). Acho que isso já dá o que pensar sobre
 aritmética modular.

 Att,
 Iuri


 On 19-11-2014 12:16, Vanderlei Nemitz wrote:

 Muito obrigado! Confesso que não entendo muito disso, mas vou procurar o
 teorema e estudar. Uma parte que não entendi bem foi:

 Observa-se que chega-se a 1 logo após a 3ª potência do 4. Além disso, a
 cada 3 potências de 4, o resto se repete. Como 31 = 1 (mod 3), temos que

 31^31 = 4^31 = 4^1 = 4 (mod 9).

 Se puder esclarecer, agradeço muito!

 Um abraço!

 Em 18 de novembro de 2014 12:25, Iuri Rezende Souza iuri_...@hotmail.com
 escreveu:

  Sim.

 A soma da soma da soma ... da soma dos algarismos de um número nos dá o
 resto do número ao ser dividido por 9.

 31 = 4 (mod 9), ou seja, 31 deixa o mesmo resto que 4 quando dividido por
 9.

 Observe o padrão do resto das potências de 4 divididas por 9:
 4^2 = 4*4 = 7 (mod 9)
 4^3 = 7*4 = 1 (mod 9)
 4^4 = 1*4 = 4 (mod 9)

 Observa-se que chega-se a 1 logo após a 3ª potência do 4. Além disso, a
 cada 3 potências de 4, o resto se repete. Como 31 = 1 (mod 3), temos que

 31^31 = 4^31 = 4^1 = 4 (mod 9).

 PS: existe um resultado em teoria dos números que diz que se mdc(a, n) =
 1, o menor inteiro não-nulo t tal que a^t = 1 (mod n) divide o número
 phi(n), onde phi(n) é o número de inteiros x menores que n tais que mdc(x,
 n) = 1. Com esse resultado, não precisa procurar padrões: basta saber que
 phi(9) = 6 e usar 31 = 1 (mod 6) a seu favor.



 On 18-11-2014 09:32, Vanderlei Nemitz wrote:

   Existe alguma maneira de resolver a questão a seguir sem precisar
 enxergar um padrão, por meio de alguns exemplos? Mesmo que esse padrão
 exista, não podemos garantir que irá permanecer. Gostaria de um método
 geral.

 Obrigado!

 *O número 31^31 é um inteiro que quando escrito na notação decimal possui
 47 **algarismos. Se a soma destes 47 algarismos é S e a soma dos
 algarismos de S **é T então a soma dos algarismos de T é igual a: *
 *a) 4 *
 *b) 5 *
 *c) 6*
 *d) 7 *
 *e) 8*

 --
 Esta mensagem foi verificada pelo sistema de antiv�rus e
 acredita-se estar livre de perigo.



 --
 Esta mensagem foi verificada pelo sistema de antivírus e
 acredita-se estar livre de perigo.



 --
 Esta mensagem foi verificada pelo sistema de antiv�rus e
 acredita-se estar livre de perigo.



 --
 Esta mensagem foi verificada pelo sistema de antivírus e
 acredita-se estar livre de perigo.


-- 
Esta mensagem foi verificada pelo sistema de antiv�rus e
 acredita-se estar livre de perigo.



[obm-l] Re: [obm-l] Prove que n é potência de 3

2014-11-26 Por tôpico saulo nilson
 primo elevado

  2^n1(2^n1+1)=P1-1

2n1log2~log(p1-1)
2n2log2~log(p2-1)
log2+n2log3+loglog2=loglog(p1-1)
llog(p2-1)log3/2log2=loglogsqrt(p1-1)
logsqrt(p2-1)=log(logsqrt(p1-1))^log2/log3
p2=1+(logsqrt(p1-1))^log4/log3
o primeiro numero primo de potencia de 3 e 73, o segundo numero p2 sera
encontrado na aproximação de log(2^n1+1)para n1log2 substituindo p1 na
formula acima  e encontrando um primo p2 na aproximação da formula sempre
teremos que n e sempre uma potencia de 3 de acordo com o equacionamento.




  Obs: potência de 3, é um número da forma 3^x onde x é natural.

 --
 Esta mensagem foi verificada pelo sistema de antivírus e
 acredita-se estar livre de perigo.


-- 
Esta mensagem foi verificada pelo sistema de antiv�rus e
 acredita-se estar livre de perigo.



[obm-l] Re: [obm-l] Como mostrar que f(x) = sen(x^2 + 1) não é periódica?

2014-11-17 Por tôpico saulo nilson
sen(x^2+1)=sen(y^2+1)
2sen[(x-y)(x+y)]/2cos(x^2+y^2+2)/2=0
x=y
x=x+p
p=0 não e periodica pois nao existe p=!0 que anule a equação acima, que
depende de x.

2014-11-11 23:04 GMT-02:00 Amanda Merryl sc...@hotmail.com:

 Boa noite.

 Isto é um tanto intuitivo, mas como podemos mostrar de forma
 matematicamente correta que a função acima, de R em R, não é periódica?

 Obrigada.

 Amanda
 --
 Esta mensagem foi verificada pelo sistema de antivírus e
  acredita-se estar livre de perigo.


 =
 Instruções para entrar na lista, sair da lista e usar a lista em
 http://www.mat.puc-rio.br/~obmlistas/obm-l.html
 =


-- 
Esta mensagem foi verificada pelo sistema de antiv�rus e
 acredita-se estar livre de perigo.



[obm-l] Re: [obm-l] Re: [obm-l] Problema de análise

2014-11-14 Por tôpico saulo nilson
-1f(-c)f(c)1=a/b ou pertence a inteiros
m*a/b=ne/d
tomando mad=neb temos o resultado.

2014-11-12 14:59 GMT-02:00 Ralph Teixeira ralp...@gmail.com:

 Hmmm... Deu vontade de olhar para g(x)=n.ln[f(x)] + m ln[f(-x)], cuja
 derivada é g'(x)=n.f'(x)/f(x) - m. f´(-x)/f(-x). Ou seja, a condição pedida
 passaria a ser g´(c)=0.

 Como g(0)=0 independentemente de m e n, basta achar um outro ponto d onde
 g(d)=0 para usar um Rolle. Ou seja, você quer mostrar que
 h(x)=ln[f(-x)]/ln[f(x)] =-m/n para algum x=d... Isto é, você quer achar um
 racional negativo na imagem de h.

 Parece que lim(x-0) h(x) = -1? Então deve ser possível arrumar um
 intervalo qualquer em volta de x=0 onde h(x) é contínua, e portanto ela
 deve assumir outros valores racionais (se ela fosse constante, seria -1,
 também serve; o problema mesmo é se f(x)=1 em uma montanha de pontos, o que
 faz h nem existir, tem que analisar isto à parte).

 Tem um monte de furinhos nessas ideias, mas acho que dá para fechar o
 problema por esse caminho?

 Abraço,
 Ralph

 2014-11-12 0:07 GMT-02:00 Artur Costa Steiner steinerar...@gmail.com:

 Oi amigos,

 Ainda não consegui resolver este não. Alguém pode colaborar?

 Suponhamos que a função real f seja contínua e positiva em  em [-1,
 1], diferenciável em (0, 1) e que f(0) = 1. Mostre que existem c em (-1, 1)
 e inteiros positivos m e n tais que

 m f(c) f'(-c) = n f(-c) f'(c)

 Obrigado.

 Artur



 --
 Esta mensagem foi verificada pelo sistema de antivírus e
 acredita-se estar livre de perigo.



 --
 Esta mensagem foi verificada pelo sistema de antivírus e
 acredita-se estar livre de perigo.


-- 
Esta mensagem foi verificada pelo sistema de antiv�rus e
 acredita-se estar livre de perigo.



[obm-l] Re: [obm-l] Re: [obm-l] Integração

2014-11-08 Por tôpico saulo nilson
sen^3 ( x ) / [ cos^4 ( x ) ]^1/3 dx
Int (senx (1-cosx^2))/(cosx)^4/3 dx
Intsenx/(cosx)^4/3dx=-3cosx^(-1/3)
Int senx*(cosx)^(2/3)=-(cosx)^5/3
R=-(cosx)^(-1/3) +(cosx)^5/3

2014-11-08 13:39 GMT-02:00 Pacini Bores pacini.bo...@globo.com:

 Oi Daniel, tome u = cosx   e   separe  sen^3(x)dx = sen^2(x).

 Tomedu = -senx.dx ;

 faça sen^2(x) = 1 - cos^2(x) e tudo ficará com  duas integrais simples em
 u com expoentes em que as integrais ficam fáceis, ok ?

 Abraços

 Pacini

 Em 7 de novembro de 2014 22:22, Daniel Rocha daniel.rocha@gmail.com
 escreveu:

 Olá a todos,

 Eu gostaria de saber qual é o resultado da integral de sen^3 ( x ) / [
 cos^4 ( x ) ]^1/3 dx.

 Eu agradeço quem responder essa.

 Um abraço.

 --
 Esta mensagem foi verificada pelo sistema de antivírus e
 acredita-se estar livre de perigo.



 --
 Esta mensagem foi verificada pelo sistema de antivírus e
 acredita-se estar livre de perigo.


-- 
Esta mensagem foi verificada pelo sistema de antiv�rus e
 acredita-se estar livre de perigo.



Re: [obm-l] problema real - off topic

2014-11-08 Por tôpico saulo nilson
Preciso cortar chapas de
38cm x 56cm

e gostaria de saber qual dos tamanhos de chapa abaixo seria o melhor (ou
seja, menor perda)
200cm x 100cm
200cm x 120cm
300cm x 100cm
300cm x 120cm

300*120 com e melhor

2014-11-08 12:06 GMT-02:00 Rogerio Ponce abrlw...@gmail.com:

 Ola' Hermann,
 escolha uma das chapas de 120cm de largura.

 Se for a de 200cm de comprimento, a divisao do comprimento por 5 (e da
 largura por 2) gera retangulos de 40cmx60cm.
 Portanto voce obtera' 10 pedacos do tamanho desejado.

 Se for a de 300cm, a divisao do comprimento por 5 (e da largura por 3)
 tambem gera retangulos de 60cmx40cm.
 Neste caso, voce obtera' 15 pedacos do tamanho desejado.

 O rendimento e' o mesmo, e a decisao devera' ser funcao do total de
 pedacos de que voce precisa.

 Repare que, ao considerar o total, talvez mesmo uma chapa com menor
 rendimento seja mais apropriada.
 Exemplo: se voce precisa de apenas dois pedacos de 38x56, use a chapa de
 100x200.

 []'s
 Rogerio Ponce

 2014-11-07 18:54 GMT-02:00 Hermann ilhadepaqu...@bol.com.br:

  Meus amigos estou com um problema real de otimização

 Preciso cortar chapas de
  38cm x 56cm

 e gostaria de saber qual dos tamanhos de chapa abaixo seria o melhor (ou
 seja, menor perda)
 200cm x 100cm
 200cm x 120cm
 300cm x 100cm
 300cm x 120cm

 Se alguém puder me ajudar agradeço muito
 Abraços
 Hermann

 --
 Esta mensagem foi verificada pelo sistema de antivírus e
 acredita-se estar livre de perigo.



 --
 Esta mensagem foi verificada pelo sistema de antivírus e
 acredita-se estar livre de perigo.


-- 
Esta mensagem foi verificada pelo sistema de antiv�rus e
 acredita-se estar livre de perigo.



Re: [obm-l] problema real - off topic

2014-11-08 Por tôpico saulo nilson
Preciso cortar chapas de
38cm x 56cm

e gostaria de saber qual dos tamanhos de chapa abaixo seria o melhor (ou
seja, menor perda)
200cm x 100cm
200cm x 120cm
300cm x 100cm
300cm x 120cm

168*76 sobra 24*168+100*32=7232cm^2
168 *114 sobra 6*168+120*32=4848
 280*76 sobra 20*100+24*280=8720
280*114 sobra 120*20+280*6=4080
300 *120 e melhor pois tem menos perda

2014-11-08 12:06 GMT-02:00 Rogerio Ponce abrlw...@gmail.com:

 Ola' Hermann,
 escolha uma das chapas de 120cm de largura.

 Se for a de 200cm de comprimento, a divisao do comprimento por 5 (e da
 largura por 2) gera retangulos de 40cmx60cm.
 Portanto voce obtera' 10 pedacos do tamanho desejado.

 Se for a de 300cm, a divisao do comprimento por 5 (e da largura por 3)
 tambem gera retangulos de 60cmx40cm.
 Neste caso, voce obtera' 15 pedacos do tamanho desejado.

 O rendimento e' o mesmo, e a decisao devera' ser funcao do total de
 pedacos de que voce precisa.

 Repare que, ao considerar o total, talvez mesmo uma chapa com menor
 rendimento seja mais apropriada.
 Exemplo: se voce precisa de apenas dois pedacos de 38x56, use a chapa de
 100x200.

 []'s
 Rogerio Ponce

 2014-11-07 18:54 GMT-02:00 Hermann ilhadepaqu...@bol.com.br:

  Meus amigos estou com um problema real de otimização

 Preciso cortar chapas de
  38cm x 56cm

 e gostaria de saber qual dos tamanhos de chapa abaixo seria o melhor (ou
 seja, menor perda)
 200cm x 100cm
 200cm x 120cm
 300cm x 100cm
 300cm x 120cm

 Se alguém puder me ajudar agradeço muito
 Abraços
 Hermann

 --
 Esta mensagem foi verificada pelo sistema de antivírus e
 acredita-se estar livre de perigo.



 --
 Esta mensagem foi verificada pelo sistema de antivírus e
 acredita-se estar livre de perigo.


-- 
Esta mensagem foi verificada pelo sistema de antiv�rus e
 acredita-se estar livre de perigo.



Re: [obm-l] Problema de pilhas

2014-11-06 Por tôpico saulo nilson
k(1+(k-1)r+1)/2=900
rk^2+k(2-r)-1800=0
delta=(2-r)^2+r7200
r=2 o menor r
k=30

2014-11-02 14:08 GMT-02:00 Mariana Groff bigolingroff.mari...@gmail.com:

 Boa Tarde,
 Alguém poderia, por favor, me auxiliar neste problema?

 Devemos distribuir 900 pedras em k pilhas, de modo que sejam satisfeitas
 as condições a seguir:
 (i) todas as pilhas têm quantidades distintas de pedras;
 (ii) se dividirmos uma das pilhas em duas pilhas não vazias, as k+1 pilhas
 resultantes não mais terão quantidades distintas de pedras.
 Ache o menor valor possível de k.

 Obrigada,
 Mariana

 --
 Esta mensagem foi verificada pelo sistema de antivírus e
 acredita-se estar livre de perigo.

-- 
Esta mensagem foi verificada pelo sistema de antiv�rus e
 acredita-se estar livre de perigo.



Re: [obm-l] Problema de pilhas

2014-11-06 Por tôpico saulo nilson
k=1
450,450

2014-11-02 14:08 GMT-02:00 Mariana Groff bigolingroff.mari...@gmail.com:

 Boa Tarde,
 Alguém poderia, por favor, me auxiliar neste problema?

 Devemos distribuir 900 pedras em k pilhas, de modo que sejam satisfeitas
 as condições a seguir:
 (i) todas as pilhas têm quantidades distintas de pedras;
 (ii) se dividirmos uma das pilhas em duas pilhas não vazias, as k+1 pilhas
 resultantes não mais terão quantidades distintas de pedras.
 Ache o menor valor possível de k.

 Obrigada,
 Mariana

 --
 Esta mensagem foi verificada pelo sistema de antivírus e
 acredita-se estar livre de perigo.

-- 
Esta mensagem foi verificada pelo sistema de antiv�rus e
 acredita-se estar livre de perigo.



[obm-l] Re: [obm-l] Convergência/divergência de uma sequência e de uma série

2014-10-31 Por tôpico saulo nilson
b1)
soma 1/(1/1+1/2^2+...+1/(n+1)^2)(n+1)^2*1 /sn/n^2=
=soma n!^2/(n-1)!^2 *n^(n-1)/n^n=soma n^(n+1)/n^n=divergente
b2) divergente


2014-10-29 22:51 GMT-02:00 Amanda Merryl sc...@hotmail.com:

 Boa noite. Estou com alguma dificuldade nisto. Agradeço se puderem ajudar
 em um deles.

 a) Seja f:[1, oo) decrescente e limitada e seja (a_n) dada por

 a_n = Soma(k = 1, n) f(k) - Int [1, n] f(x) dx, n = 1, 2,3 .

 Mostre que (a_n) converge (mesmo que a série e a integral divirjam. Em
 caso de convergência de ambas, o resultado é imediato. Aliás, pelo teste da
 integral, ou ambas convergem ou ambas divergem)

 b) Seja (a_n) uma sequência de reais positivos e (s_n) a sequência de suas
 somas parciais. Estude a convergência/divergência de Soma (a_n)/(s_n) para
 os seguintes casos:

 b.1) a_n = 1/n^2, n = 1, 2, 3

 b.2) a_n = 1/(p_n), sendo p_n o ngésimo primo.

 Muito obrigada

 Amanda.



 --
 Esta mensagem foi verificada pelo sistema de antivírus e
  acredita-se estar livre de perigo.


 =
 Instru�ões para entrar na lista, sair da lista e usar a lista em
 http://www.mat.puc-rio.br/~obmlistas/obm-l.html
 =


-- 
Esta mensagem foi verificada pelo sistema de antiv�rus e
 acredita-se estar livre de perigo.



[obm-l] Re: [obm-l] Problema de Álgebra

2014-10-29 Por tôpico saulo nilson
a) 729
b) 9216=(96)^2
94^2=8836
tem mais de uma manneira se n12


2014-10-29 18:56 GMT-02:00 Mariana Groff bigolingroff.mari...@gmail.com:

 Boa tarde,
 Não consigo resolver o problema a seguir, alguém poderia me ajudar?

 O inteiro n é o produto de dois inteiros positivos. Prove que

 (a) é possível escrever dois algarismos após os algarismos das unidades
 deste número de modo que o inteiro resultante seja um quadrado perfeito.

 (b) se n12 então só existe uma maneia de escolher estes algarismos.

 Obrigada!



 --
 Esta mensagem foi verificada pelo sistema de antivírus e
 acredita-se estar livre de perigo.

-- 
Esta mensagem foi verificada pelo sistema de antiv�rus e
 acredita-se estar livre de perigo.



Re: [obm-l] ajuda para atacar este problema

2014-10-28 Por tôpico saulo nilson
|qα − p| ≥ b/qγ
|qa| +|p|=b/q^y
|qa|=(|p|q^y-b)/q^y
|ma|=(mN^y-b)/N^y
xN==1-b/N^y  pertence [0,1]
y=1-b/N^y-1/N
teremos
|x-y|1/N

2014-10-28 17:05 GMT-02:00 Bruno Rodrigues brunorodrigues@gmail.com:

 Oi pessoal,estou sem ideias para este problema:

 Considere um número real α e constantes b  0 e γ ≥ 1 tais que para
 quaisquer p e q inteiros com q ≥ 1 vale
 |qα − p| ≥ b/qγ.
 Prove que existe uma constante C tal que, para todo inteiro N ≥ 1, o
 conjunto

 XN = {mα − ɭmα⌡, m ∈ Z, 0 ≤ m ≤ CNγ}
 é tal que, para todo x ∈ [0, 1] existe y ∈ XN com |x − y|  1/N.

 nota: ɭmα⌡ é a parte inteira de mα.

 Alguem tem alguma sugestao de como desenvolver uma bom raciocinio para ela?
 Como voces a atacariam?

 Abraços


 --
 Esta mensagem foi verificada pelo sistema de antivírus e
 acredita-se estar livre de perigo.

-- 
Esta mensagem foi verificada pelo sistema de antiv�rus e
 acredita-se estar livre de perigo.



[obm-l] Re: [obm-l] Re: [obm-l] Soluções

2014-09-26 Por tôpico saulo nilson
4) Seja X={n^3 + 3(n^2) + 3n com n igual ou maior que 0} e Y={3n - 1 com
 n0}. Prove que X=Y.

n=1
x=7
y= 2
x!=y
n=n
x-y=n^3+3n^2+3n-3n+1=n^3+3n^2+1=!0
n=n+1
x-y=n^3+3n^2+1=(n+1)^3+3(n+1)^2+1=!0


2014-09-20 21:40 GMT-03:00 Raphael Feijao raphaelfei...@hotmail.com:

 2) 5^n -1 é divisivel por 4
 passo 1) p/ n=1 - 5^1 - 1 = 4
 passo 2) para n=p -
 5^p -1 = 0 (mod 4)
 5^(p+1) = 5 (mod 4)
 5^(p+1) = 1 (mod 4)
 5^(p+1) -1 = 0 (mod 4)

 Raphael Feijão

 Em 20/09/2014, às 20:30, saulo nilson saulo.nil...@gmail.com escreveu:

 1) Prove por indução que 1 + 2^n  3^n, para n igual ou maior que 2.
 para n=2
 1+2^2=53^2
 para n=p
 3^n=(1+2)^n=1+2^n+soma(p=1 a n-1)2^p=1+2^n+k1+2^n
 para n=n+1
 1+2^(n+1)^3^n+2^n3^n+2*3^n3^(n+1)

 2014-09-20 18:23 GMT-03:00 Daniel Rocha daniel.rocha@gmail.com:

 Olá amigos,
 Â
 Eu gostaria de, POR FAVOR, obter as soluções das seguintes questões:
 Â
 1) Prove por indução que 1 + 2^n  3^n, para n igual ou maior que 2.
 Â
 2) Prove por indução que 5^n - 1 é divisível por 4, para
 n=1,2,3,4,.
 Â
 3) Prove por indução em n que o conjunto de palavras (a + ab)^n, para
 n=1,2,3,4,. é formado por todas as palavras que começam com a e não
 tem b's consecutivos.
 Â
 4) Seja X={n^3 + 3(n^2) + 3n com n igual ou maior que 0} e Y={3n - 1 com
 n0}. Prove que X=Y.
 Â
 5) Quem tem mais elementos, o conjunto dos números pares, ou o conjunto
 dos números ímpares? Justifique.
 Â
 Pessoal, essas são as questões.
 Â
 Eu aguardo sua resposta.
 Um abraço.

 --
 Esta mensagem foi verificada pelo sistema de antivírus e
 acredita-se estar livre de perigo.



 --
 Esta mensagem foi verificada pelo sistema de antivírus e
 acredita-se estar livre de perigo.


 --
 Esta mensagem foi verificada pelo sistema de antivírus e
 acredita-se estar livre de perigo.


-- 
Esta mensagem foi verificada pelo sistema de antiv�rus e
 acredita-se estar livre de perigo.



[obm-l] Re: [obm-l] Quadrado mágico(?)

2014-09-24 Por tôpico saulo nilson
x   w  a/xw

   y 15 (a/15y)

   z (a/15w)  15 w/z
x15^2w=az
z15=xw
a=15^3
a =xyz=15^3=3^3*5^3
w=1
z=3
x=45
y=25
45   175
25   159
3 125   5

uma das soluções

2014-09-22 7:43 GMT-03:00 marcone augusto araújo borges 
marconeborge...@hotmail.com:

 Completar o quadrado com números inteiros positivos de maneira que o
 resultado da multiplicação
 dos números em cada linha, coluna ou diagonal seja o mesmo

 Um quadrado 3 x 3 e só é dado o termo central 15

 --
 Esta mensagem foi verificada pelo sistema de antivírus e
 acredita-se estar livre de perigo.


-- 
Esta mensagem foi verificada pelo sistema de antiv�rus e
 acredita-se estar livre de perigo.



[obm-l] Re: [obm-l] Re: Re: Re: Função

2014-09-21 Por tôpico saulo nilson
Seja  f: R -- R , uma função definida por :
   (x+a)/(x+b) , sex  é diferente de  -b
f(x) =
   -1   , se  x  é igual a  -b

Se  f(f(x)) = x  , para todo x  real , encontre o valor de   ab .
f(1)=(a+1)/(1+b)
1=((a+1)/(1+b)+a)/((a+1)/(b+1)+b)
1=(a+1+a+ab)/(a+1+b^2+b)
-1=(a-1+ab-a)/(a-1+b^2-b)
2+2b=2a+2ab
1+b=a+ab
0=(a+ab)/(a+b^2)
a(1+b)=0
a=0
b=-1





2014-09-19 0:04 GMT-03:00 Listeiro 037 listeiro_...@yahoo.com.br:


 Eu fui direto ao cálculo de f(f(x)) = x. Nisto

 (((x+a)/(x+b))+a)/(((x+a)/(x+b))+b)=x, substituições sucessivas.

 Fiz sem levar em conta o f(-b) = -1.

 Existe uma teoria que usa uma notação matricial em expressões do tipo
 (ax+b)/(cx+d), melhor (az+b)/(cz+d), que embora o contexto seja de
 números complexos, dá certo usar produtos sucessivos de matrizes no
 caso de em (ax+b)/(cx+d) querer substituir x por (a'x'+b')/(c'x'+d').
 Com x diferente de (-d/c) e x' diferente de (-d'/c'), que é onde o
 denominador se anula.

 (ax+b)/(cx+d) em forma de matriz fica

 [a b]
 [c d]


 (x+a)/(x+b) em forma de matrix fica

 [1 a]
 [1 b]

 apenas x fica

 [1 0]
 [0 1]

 que é (x+0)/(0x+1)

 Neste caso, aqui no problema proposto encontrei a=0 e b=-1, sem
 considerar f(-b).


 Em Thu, 18 Sep 2014 12:58:20 -0300
 Gabriel Lopes cronom...@gmail.com escreveu:

  Fiquei sem entender sua explicação , poderia elaborar um pouco mais?
 
  Pensei no seguinte:
 
 
  Observe que  :
 
 (x+a)/(x+a)  = 1   , se  x é
  diferente de  -b
  a  =  b   ==f (x) =
 -1  , se  x é
  igual a  -b
 
 
 
  Temos então uma contradição pois : f(f(x)) = x . Donde  a  é
  diferente de b .
 
 
  Mas :
 
  f(f(-a)) = f(0) = -a  ,  (substituindo em : (x+a)/(x+b) ) .
 
  e:
 
  f(f(-b)) = f(-1) = -b
 
 
  Donde:
 
  (a/b) = -a  , se0  é diferente de  -b
  f(0) =
 -1 = -a  , se   0 é igual a  -b .
 
 
  Portanto  :
 
  f(f(-a)).f(f(-b)) = (-a).(-b) = ab  = f(0).(-b)  ,  donde:
 
 
  ab = -a   , se  0 é diferente de  -b
 
  ab = a.0 = b = -1(-b) = 0 , se 0 é igual a  -b .
 
 
  Em 18 de setembro de 2014 06:07, Listeiro 037
  listeiro_...@yahoo.com.br escreveu:
 
  
   A função aplicada à ela mesma. Pode ser feito assim?
   Produto de duas matrizes 2x2 igualado à matriz identidade 2x2?
  
   [1 a; 1 b] [1 a; 1 b] = [1 0; 0 1]
  
   [1 a] [1 a]  [1 0]
   [1 b] [1 b]  [0 1]
  
   [1+a a+ab; 1+b a+b^2]
  
   [1+a a+ab ]
   [1+b a+b^2]
  
   Aparentemente a=0 e b=1.
  
  
   Em Wed, 17 Sep 2014 09:30:08 -0300
   Gabriel Lopes cronom...@gmail.com escreveu:
  
Seja  f: R -- R , uma função definida por :
   
   
   (x+a)/(x+b) , sex  é diferente de  -b
f(x) =
   -1   , se  x  é igual a  -b
   
   
   
Se  f(f(x)) = x  , para todo x  real , encontre o valor de   ab .
   
  
  
   --
   Esta mensagem foi verificada pelo sistema de antivírus e
acredita-se estar livre de perigo.
  
  
  
 =
   Instru�ões para entrar na lista, sair da lista e usar a lista em
   http://www.mat.puc-rio.br/~obmlistas/obm-l.html
  
 =
  
 


 --
 Encryption works. Properly implemented strong crypto systems are one of
 the few things that you can rely on. Unfortunately, endpoint security
 is so terrifically weak that NSA can frequently find ways around it. —
 Edward Snowden

 --
 Esta mensagem foi verificada pelo sistema de antivírus e
  acredita-se estar livre de perigo.


 =
 Instru�ões para entrar na lista, sair da lista e usar a lista em
 http://www.mat.puc-rio.br/~obmlistas/obm-l.html
 =


-- 
Esta mensagem foi verificada pelo sistema de antiv�rus e
 acredita-se estar livre de perigo.



[obm-l] Re: [obm-l] Soluções

2014-09-20 Por tôpico saulo nilson
1) Prove por indução que 1 + 2^n  3^n, para n igual ou maior que 2.
para n=2
1+2^2=53^2
para n=p
3^n=(1+2)^n=1+2^n+soma(p=1 a n-1)2^p=1+2^n+k1+2^n
para n=n+1
1+2^(n+1)^3^n+2^n3^n+2*3^n3^(n+1)

2014-09-20 18:23 GMT-03:00 Daniel Rocha daniel.rocha@gmail.com:

 Olá amigos,

 Eu gostaria de, POR FAVOR, obter as soluções das seguintes questões:

 1) Prove por indução que 1 + 2^n  3^n, para n igual ou maior que 2.

 2) Prove por indução que 5^n - 1 é divisível por 4, para n=1,2,3,4,.

 3) Prove por indução em n que o conjunto de palavras (a + ab)^n, para
 n=1,2,3,4,. é formado por todas as palavras que começam com a e não tem
 b's consecutivos.

 4) Seja X={n^3 + 3(n^2) + 3n com n igual ou maior que 0} e Y={3n - 1 com
 n0}. Prove que X=Y.

 5) Quem tem mais elementos, o conjunto dos números pares, ou o conjunto
 dos números ímpares? Justifique.

 Pessoal, essas são as questões.

 Eu aguardo sua resposta.
 Um abraço.

 --
 Esta mensagem foi verificada pelo sistema de antivírus e
 acredita-se estar livre de perigo.

-- 
Esta mensagem foi verificada pelo sistema de antiv�rus e
 acredita-se estar livre de perigo.



Re: [obm-l] Equacao funcional.

2014-08-27 Por tôpico saulo nilson
f(x^2+y)+f(f(x)-y)=2f(f(x))+2y^2
y=0
f(x^2)=f(f(x))
f(x)=0
f(x^2+y)+f(-y)=2f(0)+2y^2
y=0
f(0)=f(x^2)
x^2=0
x=0 e raiz
f(0)=0
f(1)=1
f(x^2+x)+f(f(x)-x)=2ff(x)+2x^2
f(4)+f(f(2)-2)=2ff(2)+8
f(2)+f(f(1)-1)=2ff(1)+2
f(2)=4
f(4)=4+2f(4)
f(4)=-4
f(3)+f(f(2)+1)=2ff(2)+2
f(3)+f(5)=-6
f(y)+f(-y)=2y^2
f(-1)=1
1+f(3)=4
f(3)=-3
f(5)=-3
f(6)=-4
f(x^2+y)+f(f(x)-y)=2f(f(x))+2y^2
f(7)+1=-8+18
f(7)=9
f(8)=0
f(9)=41
f(10)=4
f(11)+162-41=4
f(11)=-117
e so encontrar varios pontos, plotar e encontrar as funções que se adaptam
melhor aos pontos.



2014-08-26 22:42 GMT-03:00 Jeferson Almir jefersonram...@gmail.com:

 Aproveitando o momento alguém poderia me ajudar nessa questão??

 Determine todas as funções contínuas que projeta três termos sucessivos de
 uma progressão aritmética em três termos de uma progressão geométrica.
 Desde já agradeço qualquer ajuda.


 Em 26 de agosto de 2014 07:40, Douglas Oliveira de Lima 
 profdouglaso.del...@gmail.com escreveu:

 Espetaculo, muito obrigado!!


 Em 26 de agosto de 2014 05:26, g...@impa.br escreveu:

Caro Douglas,
Fazendo y=f(x): f(x^2+f(x))+f(0)=2f(f(x))+2f(x)^2.
Fazendo y=-x^2: f(0)+f(f(x)+x^2)=2f(f(x))+2x^4.
Comparando, temos f(x)^2=x^4, donde, para todo x, f(x)=x^2 ou
 f(x)=-x^2. Em particular, f(0)=0. Fazendo então x=0 temos f(y)+f(-y)=2y^2,
 mas f(y) e f(-y) pertencem a {y^2, -y^2}, donde necessariamente
 f(y)=f(-y)=y^2. Assim, f(x)=x^2 para todo x real.
Abraços,
  Gugu


 Quoting Douglas Oliveira de Lima profdouglaso.del...@gmail.com:

  Caos amigos preciso de uma ajuda na seguinte questão, desde ja
 agradeço!!

 Problema:  Se f(x^2+y)+f(f(x)-y)=2f(f(x))+2y^2 para todos x,t
 pertencentes
 aos reais, determinar todas as funções f:R-R.

 Douglas Oliveira.

 --
 Esta mensagem foi verificada pelo sistema de antiv�rus e
  acredita-se estar livre de perigo.





 
 This message was sent using IMP, the Internet Messaging Program.



 --
 Esta mensagem foi verificada pelo sistema de antivírus e
 acredita-se estar livre de perigo.


 
 =
 Instru�ões para entrar na lista, sair da lista e usar a lista em
 http://www.mat.puc-rio.br/~obmlistas/obm-l.html
 
 =



 --
 Esta mensagem foi verificada pelo sistema de antivírus e
 acredita-se estar livre de perigo.



 --
 Esta mensagem foi verificada pelo sistema de antivírus e
 acredita-se estar livre de perigo.


-- 
Esta mensagem foi verificada pelo sistema de antiv�rus e
 acredita-se estar livre de perigo.



Re: [obm-l] inteiros

2014-08-23 Por tôpico saulo nilson
1)Sejam m e n números naturais tais que A = [(m+3)^n + 1]/3m é inteiro.
Mostre que A é impar,

3A=[C(N,0)m^n3^0+C(n,1)m^(n-1)3^1+...+c(n,n-2)m^23^(n-2)+c(n,n-1)m*3^(n-1)+c(n,n)3^n+1]/m=
=3Q+(m^n+3^n+1)/m
Para A ser inteiro
(m^n+3^n+1)/m=m^(n-1)+(3^n+1)/m tem que ser inteiro multiplo de 3
m^(n-1)=3^k-1
3A=3Q+(3^k+3^n)/m=
m tem que ser par
3Q+3^n(3^(n-k)+1)/m
3^(n-k)+1=0modm
3Q+3^n*x=3(Q+3^(n-1)x)
se x e par A e par
se x e impar A e impar
m^(n-1)=3^k-1
3^(n-k)+1/m=x
3^n/3^k=(-1+xm)/m
3^k=3^nm/(xm-1)=m^(n-1)+1
xm=m3^n/(m^(n-1)+1)+1
x e impar
logo A e impar




2014-08-22 10:19 GMT-03:00 Douglas Oliveira de Lima 
profdouglaso.del...@gmail.com:

 Para a letra b a questão foi da IMO de 1990.
 Vou dividir em duas partes:
 Parte I

 1)Como o numerador é ímpar, n deve ser ímpar.

 2)Agora vamos supor que 3^k divide n, ou seja 3^k
 é a maior potência de 3 que divide n.

  3)Assim 3^2k divide n^2 que por sua vez divide (2^n + 1).

 4)Logo 2^n é congruente a  -1 módulo 3^2k, ou seja 2^n= -1 mod(3^2k).

 5)Assim 3^(2k-1) divide n, [Pois 2 é uma raiz primitiva mod(3^n), e se
 2^n= -1 (mod3^2k),
 então 3^(2k-1) divide n], Ref. José P. dos Santos (Dê uma olhada em raízes
 primitivas e entenda este item 5 antes de passar para o próximo item)

 6) 2k - 1= k,  k= 1, mostrando
 que n tem, no máximo, um fator de 3. Observa-se que n = 3 é uma solução.
 (Não leia daqui pra frente se não entendeu o item 5)

 Parte II

 7) Suponha-se que n tem um factor primo maior do que 3, seja p este tal
 Primo. Então p divide  (2^n + 1), assim 2^n= -1 (mod p).

 8) Seja d da ordem de 2 modulo
 p, então 2^2n=1 (mod p), assim d divide 2n.
 (Novamente o assunto de raízes primitivas)

  9)Se d é ímpar, então d divide n, logo  2^n= 1, o que é absurdo.

 10)Se d é par, d=2t.. Então 2t divide 2n (Vide 8), logo t divide n.

 11)Temos também d divide (p - 1), ou
 2t divide (p-1). Assim 2t=p-1p,  ou t=(p-1)/2p .(Vide 7)

 12) Mas como t divide n, então t = 1 ou t = 3. Se t = 1,
 então d = 2 e 2^2=1 (mod p )  , novamente um absurdo.

 13) Se t=3, então d=6, assim e
 2^6= 1 (mod p), e por fermat, p = 7.

 14) Mas a ordem de 2 módulo 7 é 3,
 o que é estranho, uma vez mais contradição.

 15) Portanto, não existe esse tal p
 logo a solução n = 3.


 Valeu
 Abraços
 Douglas Oliveira.



 Em 16 de agosto de 2014 12:37, marcone augusto araújo borges 
 marconeborge...@hotmail.com escreveu:

  1)Sejam m e n números naturais tais que A = [(m+3)^n + 1]/3m é inteiro.
 Mostre que A é impar

 2) Determine todos os inteiros n  1 tais que (2^n + 1)/n^2 é inteiro.

 --
 Esta mensagem foi verificada pelo sistema de antivírus e
 acredita-se estar livre de perigo.



 --
 Esta mensagem foi verificada pelo sistema de antivírus e
 acredita-se estar livre de perigo.


-- 
Esta mensagem foi verificada pelo sistema de antiv�rus e
 acredita-se estar livre de perigo.



[obm-l] Re: [obm-l] Re: [obm-l] (Congruência de triângulos e quadriláteros)

2014-08-21 Por tôpico saulo nilson
(a-c)/D1=(b-x)/D2


2014-08-20 8:56 GMT-03:00 Douglas Oliveira de Lima 
profdouglaso.del...@gmail.com:

 Então Warley, quando falou de artista plástico acho que entendi o que pede
 na letra a.
 Faz assim chamando o paralelogramo de ABCD, coloque-o no R^3 e imagine que
 A=(m,n,a); B=(r,s,b); C=(p,q,c) e D=(x,y,z), no caso em questão o que voce
 quer é z em função de a, b e c, assim use segmentos orientados, como é
 paralelogramo AB=DC, assim C-D=B-A,  e substituindo os pontos terá (  ,
  ,c-z)=(  ,  ,b-a);
 perceba que não interessa abcissa e ordenada e sim as cotas, logo c-z=b-a,
 a última dimensão que você quer será z=a+c-b.

 Quanto a letra b não entendi o que quer.

 Abraços
 Douglas Oliveira.


 Em 19 de agosto de 2014 23:05, warley ferreira lulu...@yahoo.com.br
 escreveu:



 Boa tarde pessoal,
 gostaria de uma ajuda nesta questão.
 Desde já, agradeço.
 Att.
 Warley Souza

 Questão um (Congruência de triângulos e quadriláteros)

 a) Uma mesa decorativa inclinada em forma de paralelogramo foi criada por
 um artista
 plástico. A medida de um dos pés da mesa deveria ser de a cm. O segundo
 pé mediria b
 cm e o terceiro mediria c cm. Determine o tamanho x do quarto pé para que
 a mesa não
 manque.

 b) Substitua no item anterior o paralelogramo por um quadrilátero convexo
 ou não.


 --
 Esta mensagem foi verificada pelo sistema de antivírus e
 acredita-se estar livre de perigo.



 --
 Esta mensagem foi verificada pelo sistema de antivírus e
 acredita-se estar livre de perigo.


-- 
Esta mensagem foi verificada pelo sistema de antiv�rus e
 acredita-se estar livre de perigo.



Re: FW: [obm-l] Fobonacci

2014-08-19 Por tôpico saulo nilson
x^2+y^2+z^2=3xyz
x/yz+y/xz+z/xy=3
x=ayz
a+1/az^2+1/ay^2=3
a3
1/az^2+1/ay^20
1/z^2+1/y^20 impossivel
a0 impossivel
1/y^2+1/z^2=ab
a+b=3
a=0
1/y^2+1/b^2=0
(x,y,z)=(0,k1oo,k2oo), k1,k2pertence Z e soluçao
a=1
1/z^2+1/y^2=2
(y^2+z^2)/y^2z^2=2 soluçao (-1,1),(1,-1),(-1,-1),(1,1)
y=z
x^2+2y^2=3xy^2
delta=9y^4-8y^2
x=(3y^2+-ysqrt(9y^2-8))/2
9y^2-8=a^2
3y^2+ay=2k1
3ay+8=6k1-a^2
3a(y-a)=2(3k1-4)
y=2(3k1-4)/3a  +a
3k1-4=3k2a
k1=(3k2a+4)/3
 nao tem soluçao inteira
x^2+y^2+z^2=3xyz
y=z+a
x=z+b
z^2+2zb+b^2+z^2+2za+a^2+z^2=3z(z+a)(z+b)=3z(z^2+z(a+b)+ab)
3z^2+2z(a+b)+b^2+a^2=3z^3+3z^2(a+b)+3zab
3z^3+3z^2(a+b-1)+z(3ab-2a-2b)-b^2-a^2=0
se existe infinitos a,b inteiros tal que z seja inteiro entao existem
infinitos y,x inteiros
3a+3b-3=9k
a+b=3k-1
p=(3ab-2a-2b)/3-k^2/3
q=2k^3-k(3ab-2a-2b)/3-b^2/3-a^2/3
para que se tenha uma soluçao inteira um dos caminhos e
q^2*27=-4p^3
p=-3k1^2
q^2=4k1^6
q=2k1^3

-3k1^3=x-k^3/3
2k1^3==2k^3-x-b^2/3-a^2/3
-k1^3=5k^3/3-b^2/3-a^2/3
3k1^3+5k^3-b^2-a^2=0
3k1^3+5k^3=a^2+b^2, tem que provar que existem infinitas quadruplas de
numeros que satisfazem a equaçao anterior.
3k1^3=a^2
5k^3=b^2
a=3^nk1
b=5^nk
k1=3^(2n-1)
k=5^(2n-1)







2014-08-19 18:11 GMT-03:00 marcone augusto araújo borges 
marconeborge...@hotmail.com:

 Para Douglas oliveira e demais interessados

  Date: Mon, 16 Jan 2012 12:46:31 -0200
  Subject: Re: [obm-l] Fobonacci
  From: ralp...@gmail.com
  To: obm-l@mat.puc-rio.br

 
  Bom, eu não sabia disso mas agora que você falou...
 
  A recorrência que define os termos de ordem ímpar da seq. de Fibonacci
  pode ser obtida assim:
 
  F(2n+1)=F(2n)+F(2n-1)
  F(2n)=F(2n-1)+F(2n-2)
  F(2n-2)=F(2n-1)-F(2n-3) (tô fazendo um esforço para só deixar os de
  ordem ímpar do lado direito)
 
  Então
 
  F(2n+1)=3F(2n-1)-F(2n-3)
 
  Agora deixa eu ver os alegados quadrados. Seriam:
  F(n) 5F(n)^2-4
  1 1=1^2
  2 16=4^2
  5 121=11^2
  13 841=29^2
  ... ...
 
  Será que a sequencia da direita tem alguma ordem razoável... Digo,
  olhando para 1,4,11,29..., qual é a recorrência? Hmmm, parece que cada
  termo é 3 vezes o anterior menos ao anteanterior, de novo! Bom, mas
  isso tudo é chute, vamos ver se a gente consegue MOSTRAR isso.
 
  TEOREMA: Defina A(0)=1, A(1)=2 e A(n+1)=3A(n)-A(n-1) (n=2). Defina
  também B(0)=1, B(1)=4 e B(n+1)=3B(n)-B(n-1). Afirmo que:
 
  i) 5A(n)^2-4=B(n)^2
  ii) você já vai ver que preciso de algo mais aqui.
 
  Prova: i) Para n=0 e n=1 é só verificar direto. Por indução, se a
  propriedade vale para n=k e n=k-1, então:
  5A(k+1)^2-4 = 5(3A(k)-A(k-1))^2-4 = 45A(k)^2-30A(k)A(k-1)+5A(k-1)^2-4
  = (9B(k)^2+36)-30A(k)A(k-1)+B(k-1)^2
 
  Ah, droga, eu não tenho a mínima ideia do que fazer com aquele
  A(k)A(k-1)... Se fosse algo conhecido, razoável tipo, eu acho que
  a coisa toda vai dar (3B(k)-B(k-1))^2, né? Para isso valer, eu
  precisava que fosse 36-30A(k)A(k-1)=-6B(k)B(k-1), isto é, eu queria
  que fosse 5A(k)A(k-1)=B(k)B(k-1)+6... Como provar isto? Façamos por
  indução, ora! Então adicione lá no enunciado o seguinte:
 
  ii) 5A(n)A(n-1)=B(n)B(n-1)+6
 
  Esta propriedade claramente vale para n=1 e n=2. Agora o passo de
  indução (note que estou usando (i) com n=k, então a indução é feita
  com (i) e (ii) ao mesmo tempo!):
 
  5A(k+1)A(k) = 5(3A(k)-A(k-1))A(k) = 15A(k)^2-5A(k)A(k-1) =
  (3B(k)^2+12)-B(k)B(k-1)-6 =
  = 3B(k)^2-B(k)(3B(k)-B(k+1))+6 = B(k)B(k+1)+6
 
  Pronto! Este era o pedaço que faltava para terminar a indução em (i).
 Acabou!
 
  Abraço,
  Ralph
 
  P.S.: Agora, para DESCOBRIR que estes números funcionam, dê uma olhada
  na teoria de Equação de Pell, que ajuda a resolver coisas do tipo
  5n^2-4=p^2.
 
  2012/1/15 marcone augusto araújo borges marconeborge...@hotmail.com:
   Provar q a equação x^2+y^2+z^2=3xyz tem infinitas soluções inteiras.
  
   Essa questão ja foi resolvida na lista
   Um colega tentou uma soluçao diferente:
   Fez y=n e z=1,encontrando x^2 - 3nx +n^2 +1=0
   x= (3n + - raiz(5n^2 - 4))/2
   5n^2 - 4 deve ser um quadrado perfeito
   Tentei mostrar q existem infinitos valores de n para os quais 5n^2 - 4
 é um
   quadrado perfeito e não consegui
   Mas o colega me informou q para n igual aos termos de ordem impar da
   sequencia de fibonacci, a referida expressão
   é um quadrado perfeito(1,2,5,13,34,...)
   Não sabemos provar
   Alguem poderia esclarecer?
 
  =
  Instruções para entrar na lista, sair da lista e usar a lista em
  http://www.mat.puc-rio.br/~obmlistas/obm-l.html
  =

 --
 Esta mensagem foi verificada pelo sistema de antivírus e
 acredita-se estar livre de perigo.


-- 
Esta mensagem foi verificada pelo sistema de antiv�rus e
 acredita-se estar livre de perigo.



Re: [obm-l] Primos entre si

2014-08-09 Por tôpico saulo nilson
n+a=p1
n+b=p2
p2p1
e so auimentar p2 que da infinitos valores den


2014-08-09 10:25 GMT-03:00 Ralph Teixeira ralp...@gmail.com:

 Suponho que a e b sejam distintos... Entao suponho ba. Tome n=p-a, onde p
 eh um primo maior que ambos a e b.
 On Aug 8, 2014 8:01 PM, marcone augusto araújo borges 
 marconeborge...@hotmail.com wrote:

 Mostre que existem infinitos n tais que a + n e b + n são primos entre si


 --
 Esta mensagem foi verificada pelo sistema de antivírus e
 acredita-se estar livre de perigo.


 --
 Esta mensagem foi verificada pelo sistema de antivírus e
 acredita-se estar livre de perigo.


-- 
Esta mensagem foi verificada pelo sistema de antiv�rus e
 acredita-se estar livre de perigo.



[obm-l] Re: [obm-l] Re: [obm-l] Re: [obm-l] Raízes irracionais

2014-08-08 Por tôpico saulo nilson
Seja P(x) um polinômio não identicamente nulo e de coeficientes racionais,
e sejam a, m e n números racionais
— m e n são positivos e m^(1/2) e n^(1/2) são números irracionais. Sejam M
= m^(1/2) e N = n^(1/2). Pode-se então afirmar:

1) Se a + M é raiz de P(x), então a - M também o é (e com a mesma
multiplicidade).
P(x)=C(x-r1)(x-r2)(x-r3)=C(a-M-a-M)(a-M-r2)(a-M-r3)...
(a-M)^2-(2a)(a-M)+a^2-M^2=0 de grau 2 e- verdade
como
P^n(x)=CP2(x)*P^(n-2)(x) e p2(x)=0 entao
P^n(x)=0 tambem

2) Se M + N é raiz de P(x), então M - N, -M + N e -M-N também o são (e
todas as quatro raízes têm a mesma multiplicidade)


2014-08-07 20:31 GMT-03:00 Willy George Amaral Petrenko 
wgapetre...@gmail.com:

 Bem, o Bernardo já corrigiu o enunciado, então vou partir daí.

 Vc sabe álgebra avançada? Anéis, corpos, ideais, domínios euclidianos,
 anéis quociente, anéis de polinômios? Seria o ideal (pun intended) para
 entender a demonstração. Mas talvez dê para pegar a ideia sem isso, vou
 tentar ser didático.

 Um negócio sobre o conjunto dos polinômios de coeficientes racionais
 (chamado Q[x]), é que ele é um domínio euclidiano. Dentre outras
 propriedades, isso significa que vc pode fazer uma divisão de polinômios,
 chamada divisão euclidiana:

 Sejam f e g ∈ Q[x]. Então existem únicos q e r ∈ Q[x], com grau (r)  grau
 (g) tais que f = g*q + r

 Isso não é difícil de provar, dá pra fazer por indução no grau.

 Considere um polinômio do tipo g(x) = x2 - m, que não possui raiz
 racional (ou seja, as raiz dele são √m e -√m, ambas irracionais). Seja
 p(x), um polinômio racional com raiz a+b√m. Definimos f(x) = p(bx+a). É
 fácil ver que f tem √m como raiz. Além disso, ele continua sendo um
 polinômio racional. Gostaríamos de provar que f também possui -√m como
 raíz. Façamos a divisão de f por g:

 f(x) = (x2 - m)*q(x) + r(x), com grau (r)  grau (x2 - m) = 2. Ou seja, r
 é de grau no máximo 1 e portanto é da forma r(x) = cx + d. Mas f(√m) = 0:

 f(√m) = (√m2 - m)*q(√m) + r(√m) == 0 = r(√m) == c√m + d = 0. Mas lembre
 que r é um polinômio racional também, logo c = d = 0, pois caso contrário
 teríamos -d/c = √m, absurdo pois √m não é racional.

 Tudo isso significa que r(x) é identicamente nulo, ou seja, que f é
 divisível por x2 - m. Logo ele também possui -√m como raíz. Aplicando
 isso na definição de f, descobrimos que o polinômio original tem a-b√m como
 raiz, cqd.


 Para a parte 2, temos que considerar o corpo Q2 = {a√m + b, a,b ∈ Q}. Ele
 também é euclidiano. Considere os polinômios em Q2 = Q2[x]. Esse conjunto
 é uma extensão de Q[x] - de fato, todo polinômio com coeficientes racionais
 tem coeficientes da forma a√m + b com a = 0. A demonstração segue análoga
 à da parte 1, só que agora g(x) = x2 - n, que não possui raízes em Q2.
 Além disso, p(x) tem raiz a + b√m + c√n, e vc vai definir o f ∈ Q2 de
 forma que ele tenha √n como raíz, ou seja f(x) = p(a + b√m + cx).

 Obviamente o resultado pode ser facilmente extendido por indução para n
 raízes independentes


 Abç

 Willy


 2014-08-07 8:47 GMT-03:00 Bernardo Freitas Paulo da Costa 
 bernardo...@gmail.com:

 2014-08-07 7:21 GMT-03:00 Pedro Chaves brped...@hotmail.com:
  Prezados Colegas,
 
  Gostaria de saber se alguém conhece uma demonstração do teorema abaixo.
  Um abraço do Pedro Chaves!
  ___
 
  Teorema das raízes irracionais:
 
 
  Seja P(x) um polinômio não identicamente nulo e de coeficientes
 racionais, e sejam a, m e n números racionais
  — m e n são positivos e m^(1/2) e n^(1/2) são números irracionais.
 Sejam M = m^(1/2) e N = n^(1/2). Pode-se então afirmar:
 
  1) Se a + M é raiz de P(x), então a - M também o é (e com a mesma
 multiplicidade).
 
  2) Se M + N é raiz de P(x), então M - N, -M + N e -M-N também o são (e
 todas as quatro raízes têm a mesma multiplicidade).

 Essa segunda é falsa. Seja M = raiz(2) e N = 2*raiz(2), de forma que m
 = 2 e n = 8. Seja agora P(x) um polinômio com raiz M + N = 3*raiz(2).
 Basta tomar P(x) = x^2 - 18. Para este polinômio, -(M+N) também é
 raíz, mas nem M-N nem -(M-N) são.

 Para ser verdade, você precisa que M e N sejam racionalmente
 independentes, o que é (quase) o que você quer mostrar no teorema...

 Abraços,
 --
 Bernardo Freitas Paulo da Costa

 --
 Esta mensagem foi verificada pelo sistema de antivírus e
  acredita-se estar livre de perigo.


 =
 Instru�ões para entrar na lista, sair da lista e usar a lista em
 http://www.mat.puc-rio.br/~obmlistas/obm-l.html
 =



 --
 Esta mensagem foi verificada pelo sistema de antivírus e
 acredita-se estar livre de perigo.


-- 
Esta mensagem foi verificada pelo sistema de antiv�rus e
 acredita-se estar livre de perigo.



[obm-l] Re: [obm-l] Re: [obm-l] Re: [obm-l] Raízes irracionais

2014-08-08 Por tôpico saulo nilson
2)
M-N e raiz igual ao item 1
N-M e raiz igual ao item 1
(M+N)^2-0-(M+N)^2=0
-M-N e raiz tambem


2014-08-08 3:15 GMT-03:00 saulo nilson saulo.nil...@gmail.com:

 Seja P(x) um polinômio não identicamente nulo e de coeficientes racionais,
 e sejam a, m e n números racionais
 — m e n são positivos e m^(1/2) e n^(1/2) são números irracionais. Sejam M
 = m^(1/2) e N = n^(1/2). Pode-se então afirmar:

 1) Se a + M é raiz de P(x), então a - M também o é (e com a mesma
 multiplicidade).
 P(x)=C(x-r1)(x-r2)(x-r3)=C(a-M-a-M)(a-M-r2)(a-M-r3)...
 (a-M)^2-(2a)(a-M)+a^2-M^2=0 de grau 2 e- verdade
 como
 P^n(x)=CP2(x)*P^(n-2)(x) e p2(x)=0 entao
 P^n(x)=0 tambem

 2) Se M + N é raiz de P(x), então M - N, -M + N e -M-N também o são (e
 todas as quatro raízes têm a mesma multiplicidade)


 2014-08-07 20:31 GMT-03:00 Willy George Amaral Petrenko 
 wgapetre...@gmail.com:

 Bem, o Bernardo já corrigiu o enunciado, então vou partir daí.

 Vc sabe álgebra avançada? Anéis, corpos, ideais, domínios euclidianos,
 anéis quociente, anéis de polinômios? Seria o ideal (pun intended) para
 entender a demonstração. Mas talvez dê para pegar a ideia sem isso, vou
 tentar ser didático.

 Um negócio sobre o conjunto dos polinômios de coeficientes racionais
 (chamado Q[x]), é que ele é um domínio euclidiano. Dentre outras
 propriedades, isso significa que vc pode fazer uma divisão de polinômios,
 chamada divisão euclidiana:

 Sejam f e g ∈ Q[x]. Então existem únicos q e r ∈ Q[x], com grau (r) 
 grau (g) tais que f = g*q + r

 Isso não é difícil de provar, dá pra fazer por indução no grau.

 Considere um polinômio do tipo g(x) = x2 - m, que não possui raiz
 racional (ou seja, as raiz dele são √m e -√m, ambas irracionais). Seja
 p(x), um polinômio racional com raiz a+b√m. Definimos f(x) = p(bx+a). É
 fácil ver que f tem √m como raiz. Além disso, ele continua sendo um
 polinômio racional. Gostaríamos de provar que f também possui -√m como
 raíz. Façamos a divisão de f por g:

 f(x) = (x2 - m)*q(x) + r(x), com grau (r)  grau (x2 - m) = 2. Ou seja,
 r é de grau no máximo 1 e portanto é da forma r(x) = cx + d. Mas f(√m) =
 0:

 f(√m) = (√m2 - m)*q(√m) + r(√m) == 0 = r(√m) == c√m + d = 0. Mas
 lembre que r é um polinômio racional também, logo c = d = 0, pois caso
 contrário teríamos -d/c = √m, absurdo pois √m não é racional.

 Tudo isso significa que r(x) é identicamente nulo, ou seja, que f é
 divisível por x2 - m. Logo ele também possui -√m como raíz. Aplicando
 isso na definição de f, descobrimos que o polinômio original tem a-b√m como
 raiz, cqd.


 Para a parte 2, temos que considerar o corpo Q2 = {a√m + b, a,b ∈ Q}.
 Ele também é euclidiano. Considere os polinômios em Q2 = Q2[x]. Esse
 conjunto é uma extensão de Q[x] - de fato, todo polinômio com coeficientes
 racionais tem coeficientes da forma a√m + b com a = 0. A demonstração
 segue análoga à da parte 1, só que agora g(x) = x2 - n, que não possui
 raízes em Q2. Além disso, p(x) tem raiz a + b√m + c√n, e vc vai definir
 o f ∈ Q2 de forma que ele tenha √n como raíz, ou seja f(x) = p(a + b√m +
 cx).

 Obviamente o resultado pode ser facilmente extendido por indução para n
 raízes independentes


 Abç

 Willy


 2014-08-07 8:47 GMT-03:00 Bernardo Freitas Paulo da Costa 
 bernardo...@gmail.com:

 2014-08-07 7:21 GMT-03:00 Pedro Chaves brped...@hotmail.com:
  Prezados Colegas,
 
  Gostaria de saber se alguém conhece uma demonstração do teorema abaixo.
  Um abraço do Pedro Chaves!
  ___
 
  Teorema das raízes irracionais:
 
 
  Seja P(x) um polinômio não identicamente nulo e de coeficientes
 racionais, e sejam a, m e n números racionais
  — m e n são positivos e m^(1/2) e n^(1/2) são números irracionais.
 Sejam M = m^(1/2) e N = n^(1/2). Pode-se então afirmar:
 
  1) Se a + M é raiz de P(x), então a - M também o é (e com a mesma
 multiplicidade).
 
  2) Se M + N é raiz de P(x), então M - N, -M + N e -M-N também o são (e
 todas as quatro raízes têm a mesma multiplicidade).

 Essa segunda é falsa. Seja M = raiz(2) e N = 2*raiz(2), de forma que m
 = 2 e n = 8. Seja agora P(x) um polinômio com raiz M + N = 3*raiz(2).
 Basta tomar P(x) = x^2 - 18. Para este polinômio, -(M+N) também é
 raíz, mas nem M-N nem -(M-N) são.

 Para ser verdade, você precisa que M e N sejam racionalmente
 independentes, o que é (quase) o que você quer mostrar no teorema...

 Abraços,
 --
 Bernardo Freitas Paulo da Costa

 --
 Esta mensagem foi verificada pelo sistema de antivírus e
  acredita-se estar livre de perigo.


 =
 Instru�ões para entrar na lista, sair da lista e usar a lista em
 http://www.mat.puc-rio.br/~obmlistas/obm-l.html
 =



 --
 Esta mensagem foi verificada pelo sistema de antivírus e
 acredita-se estar livre de perigo.




-- 
Esta mensagem foi verificada pelo sistema de antiv�rus e
 acredita-se estar livre de perigo.



Re: [obm-l] Produto de cossenos

2014-08-07 Por tôpico saulo nilson
1/(2^44sen1)


2014-08-08 1:43 GMT-03:00 Walter Tadeu Nogueira da Silveira 
wtade...@gmail.com:

 Sim. Queria um outra solução sem o algebrismo puramente trigonométrico.
 Muito obrigado, Bernardo.
 Em 08/08/2014 00:38, Bernardo Freitas Paulo da Costa 
 bernardo...@gmail.com escreveu:

 2014-08-07 18:28 GMT-03:00 Walter Tadeu Nogueira da Silveira
 wtade...@gmail.com:
  Boa noite a todos.
 
  Gostaria de uma ajuda.
 
  Para calcular o produto cos1º.cos2ºcos45º é possível utilizar
 complexos
  assim: (e^i).(e^2i)...(e^45i) = e^(1+2+...45)i e tomar a parte real?
 Não. Veja que nem com apenas dois ângulos de 60° isso dá certo...
 cos(60°) = 1/2
 cos(60°)^2 = 1/4
 Re(e^(60°+60°)i) = Re(e^120°i) = cos(120°) = -1/2

 Eu imagino que foi isso que você quis dizer com tomar a parte real,
 porque a interpretação literal, ou seja, sem usar os ° para
 converter, ia dar errado já para UM ângulo, afinal, cos(45°) = exp (45
 * (pi/180) * i) != Re(exp(45i))
  Obrigado
 
  --
  Walter Tadeu Nogueira da Silveira

 Abraços,
 --
 Bernardo Freitas Paulo da Costa

 --
 Esta mensagem foi verificada pelo sistema de antivírus e
  acredita-se estar livre de perigo.


 =
 Instru�ões para entrar na lista, sair da lista e usar a lista em
 http://www.mat.puc-rio.br/~obmlistas/obm-l.html
 =


 --
 Esta mensagem foi verificada pelo sistema de antivírus e
 acredita-se estar livre de perigo.


-- 
Esta mensagem foi verificada pelo sistema de antiv�rus e
 acredita-se estar livre de perigo.



Re: [obm-l] Mais uma de diferenciabilidade

2014-07-05 Por tôpico saulo nilson
esse problema e semlhante ao anterior.


2014-07-05 0:25 GMT-03:00 Ralph Teixeira ralp...@gmail.com:

 Estou pensando em algo com o seguinte espirito (mas tem que examinar
 todos os detalhes e ver se funciona mesmo)!

 1. Suponha que f'(a) NAO EH L. Entao existe alguma sequencia (que,
 passando uma subsequencia se necessario, pode ser tomada monotona --
 vou supor spdg decrescente) z_n - a (com z_n a) tal que lim
 (f(z_n)-f(a)) / (z_n-a) nao eh L.
 2. Se a sequencia dos numeros (f(z_n)-f(a))/(z_n-a) for ilimitada,
 passe outra sub para que ele o limite dela seja +Inf ou -Inf; se for
 limitada, ela tem que ter um ponto de acumulacao que nao eh L, entao
 passe uma subsequencia para que o limite seja um numero A diferente de
 L.

 Em suma, neste momento temos uma sequencia z_n-a decrescente tal que
 lim (f(z_n)-f(a)) / (z_n-a) = A  L.

 3. Ideia: tome y_n=z_n. Agora, para CADA y_n fixo, vamos escolher x_n
 MUITO PERTO de a, tal que f(x_n) esteja MUITO MUITO PERTO de f(a).
 Assim, teremos algo do tipo
 [f(y_n) - f(x_n)]/[y_n-x_n] ~~~ [f(y_n) - f(a)]/[y_n-a] ~~~ A, que
 estaria **longe** de L. Pronto, isto seria uma contradicao frente aa
 hipotese dada!

 Vejamos os detalhes, pelo menos no caso em que A eh finito: vou
 denotar B_n=[f(y_n) - f(a)]/[y_n-a] e D=|A-L|0.
 i) Primeiro passe outra subsequencia de forma a garantir que |B_n  -
 A|  D/4. Isto eh para garantir que este negocio estah realmente longe
 de L (e eh possivel porque o limite de B_n eh A quando n-Inf, entao
 eh soh cortar o comeco da sequencia e deixar um rabo conveniente).
 ii) Agora, para um y_n fixo, note que lim (x-a) (f(y_n) - f(x))/(y_n
 - x) = B_n. Entao, para x suficientemente proximo de a, temos
 |(f(y_n)-f(x)) / (y_n-x) - B_n |D/4.
 iii) Entao escolha um x_n de cada vez, indutivamente, sempre no
 intervalo (x_(n-1),a) e satisfazendo (ii) (que simplesmente
 determinava um intervalo em volta de a onde x_n tinha que estar).
 iv) Pronto! A sequencia x_n eh crescente, e temos

 |[f(y_n) - f(x_n)]/[y_n-x_n] - L|  |A-L| - |B_n-A| - |[f(y_n) -
 f(x_n)]/[y_n-x_n] - B_n|  D - D/4 - D/4  D/2
 e portanto o limite desta fracao ali na esquerda nao serah L, absurdo.


 (Agora falta fazer um raciocinio analogo no caso em que A=+-Inf! Mas
 tenho certeza que sai, deve ateh ser mais facil do que esse que eu
 fiz.)

 Abraco, Ralph.

 2014-07-04 21:35 GMT-03:00 Merryl sc...@hotmail.com:
  Boa noite amigos
 
  Obrigada a todos pela ajuda naquele outro problema.
 
  Gostaria de ajuda com este aqui. Já pensei mas não consegui provar.
 
  Seja f:I -- R contínua no ponto a do intervalo aberto I. Suponhamos que
  para todas sequências (x_n) e (y_n) em I tais que
 
  (x_n) seja crescente e convirja para a
 
  (y_n) seja decrescente e convirja para a
 
  x_n  a  y_n para todo n
 
  exista um mesmo real L para o qual convirjam os quocientes ((f(y_n) -
  f(x_n))/(y_n - x_n)).
 
  Mostre que f é diferenciável em a e que f'(a) = L
 
  Eu tentei partir o quociente acima em duas partes, escrevendo-o como
 
  (f(y_n) - f(a))/(y_n - a) (y_n - a)/(y_n - x_n) - (f(x_n) - f(a))/(x_n -
 a)
  (x_n - a)/(y_n - x_n)
 
  Mas como a hipótese é que f é só contínua em a, não se assume
  diferenciabilidade, isto não permite chegar a f'(a) = L.
 
  Obrigada
 
  --
  Esta mensagem foi verificada pelo sistema de antivírus e
  acredita-se estar livre de perigo.

 --
 Esta mensagem foi verificada pelo sistema de antivírus e
  acredita-se estar livre de perigo.


 =
 Instru�ões para entrar na lista, sair da lista e usar a lista em
 http://www.mat.puc-rio.br/~obmlistas/obm-l.html
 =


-- 
Esta mensagem foi verificada pelo sistema de antiv�rus e
 acredita-se estar livre de perigo.



[obm-l] Re: [obm-l] Este limite é igual a f'(a) ?

2014-06-24 Por tôpico saulo nilson
u=a+h
lim(f(u+D)-f(u))/D
D=g-h
x---0 temos D---0
logo
lim (f(u+D)-f(u))/D=f´(u)=f´(a+h(0))=f´(a)


2014-06-24 1:22 GMT-03:00 Merryl sc...@hotmail.com:

 Boa noite, amigos. Gostaria de ajuda com isto,

 Seja f uma função de R em R, diferenciável em a. Sejam g e h funções
 contínuas em 0 tais que g(0) = h(0) = 0. Suponhamos que exista uma
 vizinhança deletada de 0 na qual g - h não se anule. Então, é verdade que

 lim (x -- 0) [f(a + g(x)) - f(a + h(x))]/[g(x) - h(x)]  = f'(a) ?

 Não tenho certeza. Este limite tem que existir? Se existir, é de fato
 f'(a)?

 Obrigada

 Amanda

 --
 Esta mensagem foi verificada pelo sistema de antivírus e
 acredita-se estar livre de perigo.


-- 
Esta mensagem foi verificada pelo sistema de antiv�rus e
 acredita-se estar livre de perigo.



Re: [obm-l] Limite por l'Hospital

2014-06-22 Por tôpico saulo nilson
lim (n - inf) (1+1/n)^(n²) e^(-n)
=lim(1+1/n)^n^2* e^-n
y=lim(1+1/n)^n^2
lny=limn^2ln(1+1/n) -n
lny=oo*0-oo
lny=limn(nln(1+1/n))-1)
lny=(nln(1+1/n)-1)/(1/n)
lny=(ln(1+1/n)+1/(1+n))/(-1/n^2)=0/0
lny=(-1/n*1/(n+1)-1/(n+1)^2)/2/n^3=
lny=-n^2/2(n+1)*(2n+1)/(n+1))=-limn^2(2n+1)/2(n+1)^2=-oo
y=e^-00
y=0



2014-06-23 0:43 GMT-03:00 Ralph Teixeira ralp...@gmail.com:

 Vamos ver o ln disso, que eh:

 g(x)=x^2.ln(1+1/x)-x = x^2 (ln(1+1/x)-1/x) = (ln(1+1/x)-1/x) / (x^(-2))

 Quando x-+Inf, isto aqui eh uma indet. do tipo 0/0. Note como eu
 deixei o ln o mais sozinho possivel, por que agora L'Hopital vai
 simplificar as coisas (se o ln ficar misturado com outras coisas,
 ele nao some na derivada):

 lim (x-+Inf) g(x) = lim (x-+Inf) ((-1/x^2)(1/(1+1/x))-1/x^2) /
 (-2x^(-3)) = lim (x-+inf) (-1/2)(x/(x+1)) = -1/2

 Entao, se eu nao errei conta, o limite original eh e^(-1/2).

 Abraco,
Ralph

 2014-06-23 0:17 GMT-03:00 João Maldonado joao_maldona...@hotmail.com:
  Fala galera, tem como alguém me dar uma ajuda no seguinte limite? Faz
 uma
  horta que estou tentando calcular e não sai.
 
  lim (n - inf) (1+1/n)^(n²) e^(-n)
 
  []'s
  Joao
 
  --
  Esta mensagem foi verificada pelo sistema de antivírus e
  acredita-se estar livre de perigo.

 --
 Esta mensagem foi verificada pelo sistema de antivírus e
  acredita-se estar livre de perigo.


 =
 Instruções para entrar na lista, sair da lista e usar a lista em
 http://www.mat.puc-rio.br/~obmlistas/obm-l.html
 =


-- 
Esta mensagem foi verificada pelo sistema de antiv�rus e
 acredita-se estar livre de perigo.



[obm-l] Re: [obm-l] FW: Congruência(não quero a solução)

2014-06-15 Por tôpico saulo nilson
S= 1^10 + 2^10 + ... + 100^10=
(x+y)^10=x^10+C10,1x^9y+c10.2x^8y^2+c10,3x^7y^3+c10,4x^6y^4++y^10
x^10+y^10=(x+y)^10-(x+y)f(x,y)
e x+y=101., logo S e divisivel por 101


2014-06-13 19:57 GMT-03:00 marcone augusto araújo borges 
marconeborge...@hotmail.com:

 100^10,quro dizer.

 --
 From: marconeborge...@hotmail.com
 To: obm-l@mat.puc-rio.br
 Subject: FW: Congruência(não quero a solução)
 Date: Fri, 13 Jun 2014 22:32:19 +


 A última parcela na segunda linha é 10^100,e não 10^10

 --
 From: marconeborge...@hotmail.com
 To: obm-l@mat.puc-rio.br
 Subject: Congruência(não quero a solução)
 Date: Fri, 13 Jun 2014 22:22:29 +

 Eu gostaria de alguma pista para a questão:
 Mostre que 101 divide 1^10 + 2^10 + ... + 10^10
 Se não me engano 1^100 + 2^100 + ... + 100^100 = = -1 (mod 101)
 Claro que 101 divide 1+2+...+ 100,mas...

 --
 Esta mensagem foi verificada pelo sistema de antivírus e
 acredita-se estar livre de perigo.


-- 
Esta mensagem foi verificada pelo sistema de antiv�rus e
 acredita-se estar livre de perigo.



[obm-l] Re: [obm-l] Re: [obm-l] Re: [obm-l] RE: [obm-l] Uma fórmula

2014-06-12 Por tôpico saulo nilson
1^10 + 2^10 + 3^10 + ... + 100^10
(0,999+0,001)^10+(1,999+0,001)^10++(99,999+0,001)^10~
0,999*100+99*100/2+1000*0,001=9,99+1+50*99=4960.99~4961


2014-06-10 14:58 GMT-03:00 Pedro José petroc...@gmail.com:

 Boa tarde!

 Esse somatório é complicado!

 Regis.

 log(ab) = log a + log b. Mas, log (a+b) ≠ log (ab)

 Saudações,
 PJMS


 Em 10 de junho de 2014 06:38, regis barros regisgbar...@yahoo.com.br
 escreveu:

 Caro marcone
 Pode estar errado mas os amigos da lista podem ajudar.
 log(1^10+2^10+...+100^10)=
 10log1+10log2+...+10log100=
 10(log1+log2+...+log100)=
 10(log1*2*...*100)=
 10(log100!)=
 10log(N!) onde N vai de 1 até 100
 é aqui que entra minha dificuldade.

 Regis


   Em Segunda-feira, 9 de Junho de 2014 23:30, Eduardo Henrique 
 dr.dhe...@outlook.com escreveu:


 a primeira creio que (n(n+1)/2)^10. A segunda sem ideias.

 --
 From: marconeborge...@hotmail.com
 To: obm-l@mat.puc-rio.br
 Subject: [obm-l] Uma fórmula
 Date: Tue, 10 Jun 2014 01:40:25 +

 Como calcular ( 1 + 2 + 3 + ... + 100)^10 e 1^10 + 2^10 + 3^10 + ... +
 100^10 ?

 --
 Esta mensagem foi verificada pelo sistema de antivírus e
 acredita-se estar livre de perigo.

 --
 Esta mensagem foi verificada pelo sistema de antivírus e
 acredita-se estar livre de perigo.



 --
 Esta mensagem foi verificada pelo sistema de antivírus e
 acredita-se estar livre de perigo.



 --
 Esta mensagem foi verificada pelo sistema de antivírus e
 acredita-se estar livre de perigo.


-- 
Esta mensagem foi verificada pelo sistema de antiv�rus e
 acredita-se estar livre de perigo.



Re: [obm-l] OPM 2001...

2014-05-27 Por tôpico saulo nilson
porque nao contou com os multiplos de 10, se tem o segundo algarismo que
influencia no digito nao nulo.
10*20*30*40*50=120*10^5=ultimo digito 2 nao nulo.


2014-05-27 11:03 GMT-03:00 Pedro José petroc...@gmail.com:

 Bom dia!

 Saulo,

 Na verdade você tem que retitar todos os fatores 10, ou seja dividir 50!
 por 10^m, onde 10^m || 50! ( || significa divide exatamente, não restará
 nenhum fator 10 após a divisão)
 Para 50!, m vale 12.
 Observe que não é tão períodico assim.
 Quando você faz a primeira parte, isso é os fatores Ɛ  [1,9] ᴒ |N.
 Só há um fator 10, 5 x2, portanto vai gerar: como último algarismo 8. Está
 correto.
 Porém, quando considermaos os fatores Ɛ  [11,19] ᴒ |N.
 Teremos 15 x 12 =180 tirando o fator 10, resta 8 como último algarismo.
 e ainda temos os demais fatores que nunca gerarão um produto ≡ 0 mod10.
 Portanto basta multiplicar os últimos dígitos que sobraram e o 8 obtido, ou
 seja 8*1*3*4*6*7*8*9  ≡ 4 mod10
 Considerando os fatores Ɛ  [21,29] ᴒ |N.
 Teremos apenas 25x24=600, e vai gerar 6 como último algarismo.
 Utilizando 6 e os demais fatores,que nunca gerarão um produto ≡ 0 mod10.
 Portanto basta multiplicar os últimos dígitos que sobraram e o 6 obtido, ou
 seja 6*1*2*3*6*7*8*9  ≡ 4 mod10
 Para os fatores Ɛ  [31,39] ᴒ |N. Teríamos 35*2 =70, portanto 7.
 E os restantes como já explicado, 7*1*3*4*6*7*8*9 ≡ 6 mod10

 Ou seja, a suposição que o último algarismo não nulo de cada produto
 abaixo é constante (que o levou a 8^5), está errada.

 1*2*3*4*5*6*7*8*9
 11*12*13*14*15*16*17*18*19
 21*22*23*24*25*26*27*28*29
 .
 41*41*43*44*45*46*47*48*49*

 A resposta correta é

 *dois.*
 Saudações,
 PJMS



 Em 26 de maio de 2014 15:35, saulo nilson saulo.nil...@gmail.comescreveu:

 1*2*3*4*5=20
 6*7*8*9=54
 4*2=8
 como aparece 5 vezes essa sequencia de multiplicaçoes
 8^5=8
 10*20*30*40*50=20
 20*8=160== ultimo digito 6


 2014-05-25 19:09 GMT-03:00 ruymat...@ig.com.br:

  Obrigado pela ajuda. Foi de muita valia. Abraços.




 Em 20/05/2014 12:16, terence thirteen escreveu:

 UMA coisa: um problema de uma olimpíada russa era demonstrar que este
 último dígito não forma uma sequência periódica.


 Em 20 de maio de 2014 12:16, terence thirteen 
 peterdirich...@gmail.comescreveu:

  Bem, leva um certo tempo para entender a ideia. Mas realmente não é
 complicado. O que você quer é simplesmente calcular este produto, mas sem
 levar em conta os dois e cincos nele.

 Indo devagar: imagina que você já fez essa conta (ou mandou o GNU bc ou
 o GNU Pari fazer por você), e depois fatorou essa bagaça. Se você for ver,
 o total de zeros deste numerão aí são todos provenientes dos fatores 2 e 5
 da fatoração de 50!.

 Então, fazemos assim:

 Fatoramos, e separamos os fatores 2 e 5;
 Depois, agrupamos os fatores 5 com fatores 2, de modo a formar a maior
 potência de 10 possível. Depois, descartamos essa potência sem dó!
 Os fatores ímpares que ficaram, multiplicamos seus últimos dígitos
 A potência de 2 que sobrou,

 Depois, agrupamos os fatores 5 com fatores 2, de modo a formar a maior
 potência de 10 possível. Depois, descartamos essa potência sem dó!

 Daí, os fatores 2 que sobrarem, multiplicamos!

 Vou fazer pro 20 pra tu ter uma ideia:

 01*02*03*04*05 = 8*5  * 3
 06*07*08*09*10 = 32*5* 189
 11*12*13*14*15 = 8*5  * 9009
 16*17*18*19*20 = 128*5  * 2907

 Daí fica mais fácil...




 Em 19 de maio de 2014 12:42, ruymat...@ig.com.brescreveu:

  Determinar o último algarismo  não nulo de P=1x2x3x4x5...x48x49x50.
 Eu gostaria de saber se podemos descobrir isso sem fazer multiplicações
 para cada grupo de dez números ( 1x2x3x...x10=...800;
 11x12x13...x20=...800..0; 21x22x23...x30=...200...0). Se é um exercício de
 olimpíada nível dois, fase final, acho que não deve ser feito fazendo-se
 cálculos laboriosos, ou seja, deve ter um jeito fácil. Agradeço
 antecipadamente quem resolver de um modo melhor que o exposto acima.
 Abraços. RS.

 --
 Esta mensagem foi verificada pelo sistema de antivírus e
 acredita-se estar livre de perigo.




 --
 /**/
 神が祝福

 Torres




 --
 /**/
 神が祝福

 Torres

 --
 Esta mensagem foi verificada pelo sistema de antiv?rus e
 acredita-se estar livre de perigo.


 --
 Esta mensagem foi verificada pelo sistema de antivírus e
 acredita-se estar livre de perigo.



 --
 Esta mensagem foi verificada pelo sistema de antivírus e
 acredita-se estar livre de perigo.



 --
 Esta mensagem foi verificada pelo sistema de antivírus e
 acredita-se estar livre de perigo.


-- 
Esta mensagem foi verificada pelo sistema de antiv�rus e
 acredita-se estar livre de perigo.



Re: [obm-l] OPM 2001...

2014-05-26 Por tôpico saulo nilson
1*2*3*4*5=20
6*7*8*9=54
4*2=8
como aparece 5 vezes essa sequencia de multiplicaçoes
8^5=8
10*20*30*40*50=20
20*8=160== ultimo digito 6


2014-05-25 19:09 GMT-03:00 ruymat...@ig.com.br:

  Obrigado pela ajuda. Foi de muita valia. Abraços.




 Em 20/05/2014 12:16, terence thirteen escreveu:

 UMA coisa: um problema de uma olimpíada russa era demonstrar que este
 último dígito não forma uma sequência periódica.


 Em 20 de maio de 2014 12:16, terence thirteen 
 peterdirich...@gmail.comescreveu:

  Bem, leva um certo tempo para entender a ideia. Mas realmente não é
 complicado. O que você quer é simplesmente calcular este produto, mas sem
 levar em conta os dois e cincos nele.

 Indo devagar: imagina que você já fez essa conta (ou mandou o GNU bc ou o
 GNU Pari fazer por você), e depois fatorou essa bagaça. Se você for ver, o
 total de zeros deste numerão aí são todos provenientes dos fatores 2 e 5 da
 fatoração de 50!.

 Então, fazemos assim:

 Fatoramos, e separamos os fatores 2 e 5;
 Depois, agrupamos os fatores 5 com fatores 2, de modo a formar a maior
 potência de 10 possível. Depois, descartamos essa potência sem dó!
 Os fatores ímpares que ficaram, multiplicamos seus últimos dígitos
 A potência de 2 que sobrou,

 Depois, agrupamos os fatores 5 com fatores 2, de modo a formar a maior
 potência de 10 possível. Depois, descartamos essa potência sem dó!

 Daí, os fatores 2 que sobrarem, multiplicamos!

 Vou fazer pro 20 pra tu ter uma ideia:

 01*02*03*04*05 = 8*5  * 3
 06*07*08*09*10 = 32*5* 189
 11*12*13*14*15 = 8*5  * 9009
 16*17*18*19*20 = 128*5  * 2907

 Daí fica mais fácil...




 Em 19 de maio de 2014 12:42, ruymat...@ig.com.brescreveu:

  Determinar o último algarismo  não nulo de P=1x2x3x4x5...x48x49x50. Eu
 gostaria de saber se podemos descobrir isso sem fazer multiplicações para
 cada grupo de dez números ( 1x2x3x...x10=...800; 11x12x13...x20=...800..0;
 21x22x23...x30=...200...0). Se é um exercício de olimpíada nível dois, fase
 final, acho que não deve ser feito fazendo-se cálculos laboriosos, ou seja,
 deve ter um jeito fácil. Agradeço antecipadamente quem resolver de um modo
 melhor que o exposto acima. Abraços. RS.

 --
 Esta mensagem foi verificada pelo sistema de antivírus e
 acredita-se estar livre de perigo.




 --
 /**/
 神が祝福

 Torres




 --
 /**/
 神が祝福

 Torres

 --
 Esta mensagem foi verificada pelo sistema de antiv?rus e
 acredita-se estar livre de perigo.


 --
 Esta mensagem foi verificada pelo sistema de antivírus e
 acredita-se estar livre de perigo.


-- 
Esta mensagem foi verificada pelo sistema de antiv�rus e
 acredita-se estar livre de perigo.



[obm-l] Re: [obm-l] Re: [obm-l] Re: [obm-l] Re: [obm-l] Re: [obm-l] Fwd: Mudança de base

2014-05-25 Por tôpico saulo nilson
x=-2log(2^m+34)


2014-05-25 16:57 GMT-03:00 Marcelo de Moura Costa mat.mo...@gmail.com:

 Muitíssimo obrigado a todos


 Em 24 de maio de 2014 13:33, Ralph Teixeira ralp...@gmail.com escreveu:

 Acho que o problema quer as seguintes observacoes interessantes:

 (sqrt(65)-1)(sqrt(65)+1)=65-1=64
 e
 (sqrt(65)+1)^2=66+2sqrt(65)=2(sqrt(65)+33)

 Com essas duas, tudo se arruma. Vou escrever todos os logs em base 2 (e
 nao vou escrever a base para ficar mais legivel). Entao:

 log(sqrt(65)+33)/log(sqrt(2)/2) = log((sqrt(65)+1)^2/2) / log(2^(-1/2)) =
 (2log(sqrt(65)+1) - 1 ) / (-1/2) = -4log(sqrt(65)+1) +2

 Mas

 log(sqrt(65)+1) = log(64/(sqrt(65)-1)) = 6-m

 Entao a resposta eh 4(m-6)+2=4m-22.

 Abraco, Ralph.


 2014-05-24 12:54 GMT-03:00 saulo nilson saulo.nil...@gmail.com:

 log(rq65+33)=x
 x^-1/2=rq65+33
 x^-1/2-34=rq65-1
 log2(x^-1/2-34)=m
 x=(2^m+34)^-2


 2014-05-20 23:38 GMT-03:00 terence thirteen peterdirich...@gmail.com:

 Acho que a melhor forma é simplesmente escrever $log_a(b)=ln(b)/ln(a)$.
 Isso vai te ajudar a ver o que calcular, afinal.


 Em 18 de maio de 2014 13:33, Marcelo de Moura Costa 
 mat.mo...@gmail.com escreveu:


 Alguém poderia me ajudar nesta?

 Sabe-se que:

 [image: \log_{2}{\left( \sqrt{65}-1 \right)} = m]

 Determine em função de m o valor de:

 [image: \log_{\frac{\sqrt{2}}{2}}{\left(\sqrt{65}+33\right)}]

 Que é uma mudança de base parece óbvio, mas o numerador é que está
 sendo o problema, aguardo um retorno, grato.





 --
 Esta mensagem foi verificada pelo sistema de antivírus e
 acredita-se estar livre de perigo.




 --
 /**/
 神が祝福

 Torres

 --
 Esta mensagem foi verificada pelo sistema de antivírus e
 acredita-se estar livre de perigo.



 --
 Esta mensagem foi verificada pelo sistema de antivírus e
 acredita-se estar livre de perigo.



 --
 Esta mensagem foi verificada pelo sistema de antivírus e
 acredita-se estar livre de perigo.



 --
 Esta mensagem foi verificada pelo sistema de antivírus e
 acredita-se estar livre de perigo.


-- 
Esta mensagem foi verificada pelo sistema de antiv�rus e
 acredita-se estar livre de perigo.



[obm-l] Re: [obm-l] Re: [obm-l] Fwd: Mudança de base

2014-05-24 Por tôpico saulo nilson
log(rq65+33)=x
x^-1/2=rq65+33
x^-1/2-34=rq65-1
log2(x^-1/2-34)=m
x=(2^m+34)^-2


2014-05-20 23:38 GMT-03:00 terence thirteen peterdirich...@gmail.com:

 Acho que a melhor forma é simplesmente escrever $log_a(b)=ln(b)/ln(a)$.
 Isso vai te ajudar a ver o que calcular, afinal.


 Em 18 de maio de 2014 13:33, Marcelo de Moura Costa 
 mat.mo...@gmail.comescreveu:


 Alguém poderia me ajudar nesta?

 Sabe-se que:

 [image: \log_{2}{\left( \sqrt{65}-1 \right)} = m]

 Determine em função de m o valor de:

 [image: \log_{\frac{\sqrt{2}}{2}}{\left(\sqrt{65}+33\right)}]

 Que é uma mudança de base parece óbvio, mas o numerador é que está sendo
 o problema, aguardo um retorno, grato.





 --
 Esta mensagem foi verificada pelo sistema de antivírus e
 acredita-se estar livre de perigo.




 --
 /**/
 神が祝福

 Torres

 --
 Esta mensagem foi verificada pelo sistema de antivírus e
 acredita-se estar livre de perigo.


-- 
Esta mensagem foi verificada pelo sistema de antiv�rus e
 acredita-se estar livre de perigo.



Re: [obm-l] Lista 4 Cone Sul 2008

2014-05-17 Por tôpico saulo nilson
n1!(n1!^2006-1)=f(n1)

n2!(n2!^2006-1)=f(n2)
n1=n2
f(n1)=f(n2)
n1=!n2
f(n1)=!f(n2)




2014-05-17 10:47 GMT-03:00 Gabriel Lopes cronom...@gmail.com:

 9 . Prove que a função f : N -- Z definida por :

 f(n) = (n^2007) − n!

 é injetiva.

 --
 Esta mensagem foi verificada pelo sistema de antivírus e
 acredita-se estar livre de perigo.

-- 
Esta mensagem foi verificada pelo sistema de antiv�rus e
 acredita-se estar livre de perigo.



Re: [obm-l] Mais uma que quero compartilhar!!

2014-05-15 Por tôpico saulo nilson
y=loglim fn=lim log((an n^n+an-1n^n-1++1))/2^n===0
n-oo
y=1


2014-05-15 17:45 GMT-03:00 Douglas Oliveira de Lima 
profdouglaso.del...@gmail.com:

 Essa vai em homenagem a meu grande amigo Gandhi(Antonio Luiz Santos) que
 me ensinou como fazer, quero dizer também que essa lista da obm(do qual
 usamos para discutir questões de olimpíadas e outras questões
 interessantes) esta sendo pra mim muito gratificante neste momento, porque
 nos que gostamos de matemática, gostamos de resolver questões ate em papel
 de guardanapo no restaurante,pois desde que me mudei para Brasilia não
 encontrei professores aqui como os que tive a oportunidade de  conhecer no
 Rio de Janeiro como Gandhi, Carlos Victor, Eduardo Mauro, Eduardo Wagner,
 Haroldo, Poncio, Ivan, Bandeira, e claro não poderia esquecer do grande
 Alvaro, e que me ajudaram a crescer dentro desta área. Existem outros que
 conheci, mas hoje o mérito vai pra eles,professores humildes, que nunca me
 negaram sequer uma questão, ajudando o amigo a crescer para que um dia
 possamos derrubar essa grande massa de professores ruins(que não gostam de
 estudar) do mercado,e quem sabe assim incentivando a futura geração a
 curtir matemática.

 Encontrar o valor de (1+2(1+3(1+4(1+...)^(1/2))^(1/2))^(1/2))^(1/2)

 --
 Esta mensagem foi verificada pelo sistema de antivírus e
 acredita-se estar livre de perigo.

-- 
Esta mensagem foi verificada pelo sistema de antiv�rus e
 acredita-se estar livre de perigo.



[obm-l] Re: [obm-l] Mais duas questões excelentes de geometria!!!

2014-05-15 Por tôpico saulo nilson
xc^2+yc^2=(5/12a)^2
yr=a/2
a/-a=-1=(z-a)/(x-a)
y-a=-z+a
zr+xr=2a
-axca
0zr2a
D=sqrt((xc-xr)^2+(yc-a/2)^2+(zr)^2)
=sqrt(25/144a^2-2xcxr+xr^2+-ayc+a^2/4+zr^2)2(51a^2/144
*2a^2*a^2*a^2)^1/82a(61/72)^1/8







2014-05-15 17:24 GMT-03:00 Douglas Oliveira de Lima 
profdouglaso.del...@gmail.com:

 Quando estava estudando para a prova do colégio naval em 1997, no colégio
 e curso tamandare da ilha do governador(Diretores Orozimbo e Oswaldo) me
 deparei com uma apostila em xerox escrita pelos professores Carlos Victor e
 Eduardo Mauro, que tinham provas resolvidas do colégio naval e no final da
 apostila tinham questões propostas por eles, e uma delas que gostei muito e
 fiz na época(sem uso de calculo)
  quero compartilhar com os senhores, e também outro problema numero 2 que
 ainda não consegui fazer mas estou tentando. A saber so fui conseguir a
 apostila no final do curso perto da prova assim continuei estudando ate que
 passei em primeiro lugar geral de matemática(nesta época um ano atras
 pensei que não passaria numa prova dessas nunca).

 PROBLEMA 1)(Proposta por Carlos Victos e Eduardo Mauro ) Dado um quarto de
 circulo AOB, de raio OA, prolonga-se o raio OA, e pelo ponto B, traca-se
 uma perpendicular ao raio OB.

 a) Tracar a este quarto de circulo uma tangente MN, tal que a área do
 trapézio BMNO seja igual a uma área dada m^2.
 b) Ache o minimo da area do trapezio.

 PROBLEMA 2)Seja um cubo de aresta a. Seja N um ponto na diagonal de uma
 face lateral, M um ponto no círculo que se encontra no plano da base com
 centro no centro da base e raio (5/12)a  Encontre o menor valor da medida
 do segmento MN.

 --
 Esta mensagem foi verificada pelo sistema de antivírus e
 acredita-se estar livre de perigo.

-- 
Esta mensagem foi verificada pelo sistema de antiv�rus e
 acredita-se estar livre de perigo.



Re: [obm-l] Errata

2014-05-08 Por tôpico saulo nilson
2014-04-28 11:43 GMT-03:00 Pedro José petroc...@gmail.com:

 Bom dia!

 Por intuição a ordem decrescente é assim:

 n! , (log n)^n e n^logn.

 log de n torna o expoente  n e embora a base seja bem menor no final das
 contas  o segundo termo deve ser maior que o primeiro.

 É fácil observar que: n! tem pelo menos  0,5 * n termos com valores =
 0,5 n (i) como n é muito grande é bem provável que seja o primeiro

 Porém, deveremos provar:

 Sejam a1 = n!, a2 = (logn)^n e a3 = n^logn, onde n= 2010^2010.

 Como log a x é uma função monótona crescente para a 1  temos que:

 loga  logb == ab.

 log a2 = n.log(logn)= 2010^2010*log(2010*log2010)

 log a3=(log n)^2=(2010*log2010)^2

 É fácil verificar que a2  a1.

 2010^2010*(log2010+log(log2010))  (2010*log2010)^2

 Lembrar que log 2010 Ɛ (3,4).

 Por (i) temos que: n!  (n/2)^(n/2); pois todos os fatores de n! são
 inteiros e positivos.

 Seja y= (n/2)^(n/2) == log y = (n/2). (log n – log 2) ==

 == log y = 0.5*(2010^2010)*(2010*log2010-log 2)

 log y  log a2 (ii), pois:  0.5*(2010*log2010 – log 2) 
 log2010+log(log2010)

 Atentar que (log 2010 + log(log(2010)) Ɛ (3,5)

 De (ii) temos que y  a2. Como a1  y == a1  a2.

 Portanto, em ordem decrescente n! , (log n)^n e n^logn.

 Saudações,
 PJMS


 Em 24 de abril de 2014 00:36, ruymat...@ig.com.br escreveu:

  Errata: Na verdade gostaria de colocar em ordem crescente os números:
 n^logn , n! e (logn)^n sabendo-se que n= 2010^2010. Desculpem-me. Agradeço
 antecipadamente a quem ajudar. Abraços




 --
 Esta mensagem foi verificada pelo sistema de antivírus e
 acredita-se estar livre de perigo.



 --
 Esta mensagem foi verificada pelo sistema de antivírus e
 acredita-se estar livre de perigo.


-- 
Esta mensagem foi verificada pelo sistema de antiv�rus e
 acredita-se estar livre de perigo.



[obm-l] Re: [obm-l] Soma trigonométrica

2014-05-07 Por tôpico saulo nilson
=46+d/dxtg(2x+88)(45-somatgxtg(90-x)=46



2014-05-07 8:42 GMT-03:00 Vanderlei Nemitz vanderma...@gmail.com:

 Alguém tem alguma ideia? Tentei utilizar a fórmula da tangente do arco
 duplo, mas ficou complicado.


 Mostre que *tg²(1°) + tg²(3°) + tg²(5°) + ...+ tg²(89°)*  é um número
 inteiro.


 Obrigado!

 --
 Esta mensagem foi verificada pelo sistema de antivírus e
 acredita-se estar livre de perigo.

-- 
Esta mensagem foi verificada pelo sistema de antiv�rus e
 acredita-se estar livre de perigo.



[obm-l] Re: [obm-l] Re: [obm-l] Questão da 3ª fase nível 1 da OBM 2013

2014-04-26 Por tôpico saulo nilson
222  4

1  6  12

 9  18  3


b)
8 412

 2   1 3

  105 15






2014-04-23 23:13 GMT-03:00 Willy George Amaral Petrenko 
wgapetre...@gmail.com:

 vc quer uma ajuda ou uma solução?

 Uma ajuda:

 a) Observe que 22 não tem muitos divisores próprios, apenas 1,2 e 11.
 Mostre que se 11 fizer parte do quadrado, então algum outro múltiplo de 11
 além do 22 também estará (ou seja, não existe quadrado onde os únicos
 múltiplos de 11 sejam 11 e 22). Assim se 22 for o maior, 11 não pode estar
 no quadrado.

 Portanto 22 deverá estar ao lado de 1 e 2, logo deve estar no canto:

 22  1  x
 2   x   x
 x   x   x

 Agora eu fiz por tentativa e erro.

 b) Generalize a observação que eu fiz para um primo qualquer (ou seja se p
 está no quadrado, então outro múltiplo de p que não é o 2p também estará).
 Tente achar um quadrado cujo maior número seja o menor que conseguir e use
 a observação para mostrar que não existe nenhum menor.

 Se não conseguir fazer eu posso mandar a solução.


 2014-04-23 18:37 GMT-03:00 Érica G. Pongelupe Giacoia profer...@ig.com.br
 :



 Prezados,
 Gostaria da ajuda de vcs para resolver a seguinte questão que caiu na 3ª
 fase (nível 1) da OBM do ano passado.

 Desde já obrigada a todos.

 Érica G. P. Giacoia

 (OBM 2013) Desejamos preencher um tabuleiro 3x3 com 9 inteiros positivos
 distintos sendo que números a e b que têm um lado em comum devem ser tais
 que a é divisível por b ou b é divisível por a.

 Vejamos uma configuração que satisfaz as condições do problema. Observe
 que o maior número que aparece no tabuleiro é o 25.

 8 2   10

 420   5

 12   1   25

 a) Apresente uma maneira de preencher um tabuleiro de modo que o maior
 número que aparece é  22.

 b) Qual é o menor inteiro positivo que pode ser o maior número que
 aparece no tabuleiro?



 --
 Esta mensagem foi verificada pelo sistema de antivírus e
 acredita-se estar livre de perigo.



 --
 Esta mensagem foi verificada pelo sistema de antivírus e
 acredita-se estar livre de perigo.


-- 
Esta mensagem foi verificada pelo sistema de antiv�rus e
 acredita-se estar livre de perigo.



[obm-l] Re: [obm-l] Minimizar a distância

2014-04-26 Por tôpico saulo nilson
as 3 cidades formam um triangulo, e so encontrar um ponto dentro do
triangulo q que minimize a soma das distancias.
d=200(sqrt(x^2+y^2)+2sqrt((x-xa)^2+(y-ya)^2)+3sqrt((x-xb)^2+(y-yb)^2)
tgu=(yb)/(xb-xa)
tgv=ya/xa
area do triangulo
p=seminperimetro=(d1+d2+d3)/2
S=sqrt(p(p-d1)(p-d2)(p-d3))
distancia de um ponto a reta
retas que os lados pertencem
tgv=(y)/(x-xa)
y=xtgv-xatgv
S1=L1*|xtgv-y-xatgv|2/sqrt(tgv^2+1)
S2=L2*y/2
tgu=(y)/(x-xa)
y=xtgu-xatgu
S3=L3*|xtgu-y-xatgu|/2sqrt(tgu^2+1)
L1cosv(y+xatgv-xtgv)/2+L2y/2+L3cosu(y+xatgu-xtgu)/2=S
y(L1cosv+L2+L3cosu)=2S+x(senvL1+senuL3)-xa(L1senv+L3senu)
y=a+bx
d=200(sqrt(x^2+y^2)+2sqrt((x-xa)^2+(y-ya)^2)+3sqrt((x-xb)^2+(y-yb)^2)
y=a+bx
d=200(sqrt(x^2+(a+bx)^2)+2sqrt(x-xa)^2+(a+bx)^2)+3sqrt((x-xb)^2+(a+bx-yb)^2))
dd/dx=0
(x+b(a+bx))/sqrt(x^2+(a+bx)^2)  +2(x-xa+b(a+bx))/sqrt((x-xa)^2+(a+bx)^2)
 +3(x-xb+b(a+bx-yb))/sqrt((x-xb)^2+(a+bx-yb)^2))=0


2014-04-22 3:29 GMT-03:00 Listeiro 037 listeiro_...@yahoo.com.br:



 Sem a planta de arruamento poderia ser no centro da circunferência a
 que os três pontos onde estão as escolas pertence. Se forem colineares,
 no meio do segmento de reta formado pelos dois pontos em que se
 encontram as escolas mais distantes entre si.

 É isso?


 Em Sun, 20 Apr 2014 21:06:33 +
 marcone augusto araújo borges marconeborge...@hotmail.com escreveu:

  Três cidadezinhas têm 100,200 e 300 estudantes,respectivamente.Onde
  se deve construiruma escola para minimizar a distância total
  percorrida pelos estudantes todos os dias?
 


 --
 Esta mensagem foi verificada pelo sistema de antivírus e
  acredita-se estar livre de perigo.


 =
 Instru�ões para entrar na lista, sair da lista e usar a lista em
 http://www.mat.puc-rio.br/~obmlistas/obm-l.html
 =


-- 
Esta mensagem foi verificada pelo sistema de antiv�rus e
 acredita-se estar livre de perigo.



Re: [obm-l] Quadrado perfeito ?

2014-04-12 Por tôpico saulo nilson
a00b
a=b
a(101)=nao e quadrado perfeito
a=!b
a00.b=a*10^n=(x-rqb)(x+rqb)=
=a*2^n*5^n
como x -rqb e x+rqb diferem de 2rqb e nos temos  combinaçoes que diferem de
multiplos de  2 e 5,  e b varia de 1 a 9 logo x nunca podera ser escolhido
para que a igualdade seja igualada.

2014-04-06 16:27 GMT-03:00 terence thirteen peterdirich...@gmail.com:

 Vou supor que exista pelo menos um 0.

 3*10^n+1 = x^2
 3*10^n= x^2-1
 3*10^n= (x-1)(x+1)

 3*2^n*5^n= (x-1)(x+1)

 Temos MDC(x-1,x+1)=MDC(x-1,2)=1 ou 2. Como n1, então o MDC é 2. Assim, o
 lado direito é múltiplo de 4 mas não de 8. Isso limita o total de valores
 possíveis para n - basta testar!

 Acho que dá para fazer o mesmo nos outros casos que você deixou para
 trás...






 Em 5 de abril de 2014 20:39, marcone augusto araújo borges 
 marconeborge...@hotmail.com escreveu:

  Mostre que os números da forma a000...0b não são quadrados perfeitos

 Os valores possíveis para b são 1,4,5,6 e 9
 Analisando modulo 8 descartamos 6 e 9
 Podemos descartar tambem o 5,pois se a^2 termina em 5,a tambem
 termina em 5,mas neste caso a^2 terminaria em 25
 Analisando modulo 9,notamos que 1000...01,2000...01,4000...1,5000...1 e
 7000...1 não são quadrados
 Também estariam fora 1000...04,2000...04,4000...04,7000...04,8000...4
 Os quadrados são da forma 9k,9k+1,9k+4 e 9k+7
 Há outros 8 casos que ficariam em aberto: 3000...01,6000...01,8000...01 e
 9000...01,3000...04,5000...04,
 6000...04 e 9000...04
 E agora José?



 --
 Esta mensagem foi verificada pelo sistema de antivírus e
 acredita-se estar livre de perigo.




 --
 /**/
 神が祝福

 Torres

 --
 Esta mensagem foi verificada pelo sistema de antivírus e
 acredita-se estar livre de perigo.


-- 
Esta mensagem foi verificada pelo sistema de antiv�rus e
 acredita-se estar livre de perigo.



Re: [obm-l] Quadrado perfeito?

2014-03-19 Por tôpico saulo nilson
292929292929292...2929=
=29*1010101010101010101;10101

1010101010101010101;10101 esse numero deve ser divisivel po 29 senao
nao e quadrado perfeito
101/29=3k+14
140/29=4k+24
241/29=8k+9
90/29=3k+3
31/29=k+2
201/29=6k+27
270/29=9k+9
91/29=3k+4
40/29=k+11
111/29=3k+44
440/29=15k+5
51/29=k+22
220/29=7k+17
171/29=5k+26
260/29=8k+28
281/29=8k+20
200/29=6k+26
261/29=8k+29
290/29=10k +0 aqui começa a repetir, multiplo de 22 digitos pode ser
divisivel, senao tem que continuar a dividir, supondo que tenha 22 digitos
, como termina em zero nao e quadrado perfeito pois sempre vai sobrar sqrt10
34831069313151758868103483



2014-03-18 16:26 GMT-03:00 marcone augusto araújo borges 
marconeborge...@hotmail.com:

 Que bobeira,quadrados não terminam em 7.
 Mas eu não saberia afirmar se algum número da forma 2929...29 é quadrado
 perfeito.

 --
 From: marconeborge...@hotmail.com
 To: obm-l@mat.puc-rio.br
 Subject: [obm-l] Quadrado perfeito?
 Date: Tue, 18 Mar 2014 18:07:46 +


 Números da forma 2525...25 e 1717...17 podem ser quadrados perfeitos ?
 Terence sugeriu módulo 8  para o primeiro mas eu já tinha visto que não
 serve
 No caso de 111...11,esse número deixa resto 7 quando dividido por 8 e
 nenhum
 quadrado é da forma 8k + 7.Ai serve.




 --
 Esta mensagem foi verificada pelo sistema de antivírus e
 acredita-se estar livre de perigo.

 --
 Esta mensagem foi verificada pelo sistema de antivírus e
 acredita-se estar livre de perigo.


-- 
Esta mensagem foi verificada pelo sistema de antivírus e
 acredita-se estar livre de perigo.



[obm-l] Re: [obm-l] Número de soluções naturais

2014-03-17 Por tôpico saulo nilson
x=1
y=2
z=199
x=1
y=7
z=197
pa de razao 2 em z
1=199-(n-1)2
n=100 soluçoes  para x=1
x=2
y=4
z=198
x=2
y=9
z=196
0=198-2(n-1)
n=100 soluçoes para x=2
x=3
y=1
z=199
x=3
y=6
z=197
100 soluçoes para x=3
tem que descobrir ate que valor de x temos 100 soluçoes
x=1000 uma soluçao
x=999 nao tem soluçao
x=998
y=1
z=0 uma soluçao
x=997 nao tem soluçao
x=996 uma soluçao
x=995 uma soluçao
x=994 uma soluçao
x=993 uma soluçao
x=992 uma soluçao
x=991  uma soluçao
2x+5z=10 ate 20   2 soluçoes=22 soluçoes
x=1 atte 100
100*100+1
x=2 ate 200
10*100+10*99+10*98+10*1+20=10(101)50+20=50520



2014-03-05 20:22 GMT-03:00 Ennius Lima enn...@bol.com.br:

 Caros Colegas,

 Quantas soluções naturais tem a equação diofantina x + 2y + 5z = 1000?

 (Incluo o zero entre os números naturais)


 Desde já, agradeço-lhes a atenção.

 Ennius Lima
 __-


 --
 Esta mensagem foi verificada pelo sistema de antivírus e
  acredita-se estar livre de perigo.



-- 
Esta mensagem foi verificada pelo sistema de antivírus e
 acredita-se estar livre de perigo.



Re: [obm-l] infinitas ternas

2014-02-28 Por tôpico saulo nilson
5c^2+1997=3c^2+2c^2+1997
1997=2x+3y
2(a^2-c^2)+3(b^2-c^2)=1997
2x+3y=1997
que tem infinitas soluçoes inteiras como x=2*952+3*31, o que nos leva a um
outro problema que e:
a^2-c^2=x=-d^2
b^2-c^2=y=-e^2
onde a, b e c sao inteiros o que e equivalente a encontrar infinitos
triangulos retangulos com lados inteiros, que ja foi feito aqui.
b^2-a^2=y-x
b^2+e^2=a^2+d^2=c^2
(b/c)^2+(e/c)^2=1
sena=b/c=1/2 ou 2/3 ou 5/7 que sao infinitos valores




2014-02-27 19:21 GMT-03:00 marcone augusto araújo borges 
marconeborge...@hotmail.com:

 Sejam a,b e c números inteiros positivos.Mostre que existem infinitas
 ternas (a,b,c)  que são soluções da equação 2a^2 + 3b^2 - 5c^2 = 1997

 --
 Esta mensagem foi verificada pelo sistema de antivírus e
 acredita-se estar livre de perigo.


-- 
Esta mensagem foi verificada pelo sistema de antivírus e
 acredita-se estar livre de perigo.



Re: [obm-l] infinitas ternas

2014-02-28 Por tôpico saulo nilson
(1401,2401,2060) e uma soluçao
(3249,4249,3880) e outra soluçao


2014-02-28 15:24 GMT-03:00 saulo nilson saulo.nil...@gmail.com:

 5c^2+1997=3c^2+2c^2+1997
 1997=2x+3y
 2(a^2-c^2)+3(b^2-c^2)=1997
 2x+3y=1997
 que tem infinitas soluçoes inteiras como x=2*952+3*31, o que nos leva a um
 outro problema que e:
 a^2-c^2=x=-d^2
 b^2-c^2=y=-e^2
 onde a, b e c sao inteiros o que e equivalente a encontrar infinitos
 triangulos retangulos com lados inteiros, que ja foi feito aqui.
 b^2-a^2=y-x
 b^2+e^2=a^2+d^2=c^2
 (b/c)^2+(e/c)^2=1
 sena=b/c=1/2 ou 2/3 ou 5/7 que sao infinitos valores




 2014-02-27 19:21 GMT-03:00 marcone augusto araújo borges 
 marconeborge...@hotmail.com:

  Sejam a,b e c números inteiros positivos.Mostre que existem infinitas
 ternas (a,b,c)  que são soluções da equação 2a^2 + 3b^2 - 5c^2 = 1997

 --
 Esta mensagem foi verificada pelo sistema de antivírus e
 acredita-se estar livre de perigo.




-- 
Esta mensagem foi verificada pelo sistema de antivírus e
 acredita-se estar livre de perigo.



[obm-l] Re: [obm-l] integral alguém se habilita?

2014-02-28 Por tôpico saulo nilson
I=itntegral
I (10x^2+18)/3sqrt2sqrt(x^2+2)(5x^2+9)  dx
I 10x^2/3sqrt2sqrt(5x^4+19x^2+18)+6/sqrt(2)sqrt(5x^4+19x^2+18) dx
I 6/sqrt(2)sqrt(5x^4+19x^2+18) dx
= 6/sqrt2 I 1/sqrt((sqrt5*x^2+19/2sqrt5)^2+18-(19/2sqrt5)^2)
5x^2+19/2sqrt5=u
10xdx=du
dx=du/10x
=du/10sqrt(u-19/2sqrt5)/5
=6sqrt5/10*sqrt2 * I 1/sqrt(u-19/2sqrt5)sqrt (u^2+18-(19/2sqrt5))
e catalogada em livros
vc tem que fazer a substituiçao
1/(u-19/2sqrt5)=y
que cai em outra integral catalogada







2014-02-28 14:27 GMT-03:00 Hermann ilhadepaqu...@bol.com.br:

  integrate (sqrt((10x^2+18)/(9x^2+18))) dx

 alguém saberia fazer?

 coloquei no Wolfram e me assustei, abraços Hermann

 --
 Esta mensagem foi verificada pelo sistema de antivírus e
 acredita-se estar livre de perigo.


-- 
Esta mensagem foi verificada pelo sistema de antivírus e
 acredita-se estar livre de perigo.



[obm-l] Re: [obm-l] RE: [obm-l] Re: [obm-l] Número inteiro

2014-02-26 Por tôpico saulo nilson
porque bp e o maior numero k=ab por isso apareceu 2ab, b=a porque p tem
quer ser primo e inteiro primeiro.


2014-02-25 21:57 GMT-03:00 marcone augusto araújo borges 
marconeborge...@hotmail.com:

 Olá,Saulo
 Eu agradeceria muito se vc detalhasse mais o seu pensamento.
 Por exmplo,por que k+a = bp?
 Por que não k-a = bp?
 Como apareceu 2ab?
 Vc considerou b = a ou b = ac? Por que?


 --
 Date: Tue, 25 Feb 2014 15:26:14 -0300
 Subject: [obm-l] Re: [obm-l] Número inteiro
 From: saulo.nil...@gmail.com
 To: obm-l@mat.puc-rio.br


 k^2-kp=n^2
 (k-n)(k+n)=kp
 k-n=a
 k+n=bp
 2ab=a+bp
 p=a(2b-1)/b
 b=a
 p=2a-1 infinitas soluçoes
 b=ac
 p=(2ac-1)/c
 2xc+c=2ac-1
 1+c=2c(a-x)
 impossivel pois 2c1+c


 2014-02-25 7:06 GMT-03:00 marcone augusto araújo borges 
 marconeborge...@hotmail.com:

 Seja p um primo ímpar dado.Para exatamente quantos valores de k inteiro
 positivo
 (k^2 - kp)^1/2 é também inteiro?

 --
 Esta mensagem foi verificada pelo sistema de antivírus e
 acredita-se estar livre de perigo.



 --
 Esta mensagem foi verificada pelo sistema de antivírus e
 acredita-se estar livre de perigo.

 --
 Esta mensagem foi verificada pelo sistema de antivírus e
 acredita-se estar livre de perigo.


-- 
Esta mensagem foi verificada pelo sistema de antivírus e
 acredita-se estar livre de perigo.



[obm-l] Re: [obm-l] Número inteiro

2014-02-25 Por tôpico saulo nilson
k^2-kp=n^2
(k-n)(k+n)=kp
k-n=a
k+n=bp
2ab=a+bp
p=a(2b-1)/b
b=a
p=2a-1 infinitas soluçoes
b=ac
p=(2ac-1)/c
2xc+c=2ac-1
1+c=2c(a-x)
impossivel pois 2c1+c


2014-02-25 7:06 GMT-03:00 marcone augusto araújo borges 
marconeborge...@hotmail.com:

 Seja p um primo ímpar dado.Para exatamente quantos valores de k inteiro
 positivo
 (k^2 - kp)^1/2 é também inteiro?

 --
 Esta mensagem foi verificada pelo sistema de antivírus e
 acredita-se estar livre de perigo.


-- 
Esta mensagem foi verificada pelo sistema de antivírus e
 acredita-se estar livre de perigo.



[obm-l] Re: [obm-l] Três de inteiros

2014-02-20 Por tôpico saulo nilson
1--
x^3+y^3=(x+y)(x^2-xy+y^2)=(x+y)^2
x=-y
ou
x^2-xy+y^2-x-y=0
delta=(1+y)^2-4y^2+4y=1+2y+y^2-4y^2+4y=1+6y-3y^2
x=(1+y+-sqrt(4-3(y-1)^2))/-6nao serve pois nao tem soluçoes inteiras
2--
m+n=33
3m^2n+3mn^2=99mn





2014-02-20 11:23 GMT-03:00 marcone augusto araújo borges 
marconeborge...@hotmail.com:

 1) Ache todos os pares de inteiros (x,y) tais que x^3 + y^3 = (x + y)^2

 2) Determine todos os pares de inteiros (m,n) tais que m.n  = 0 e
 m^3 + n^3 + 99mn = 33^3

 3) Se x,y,z são números reais não nulos,com x+y+z também não nulo
 Calcule os valores possíveis da expressão (x^2 + y^2 + z^2)/2(xy+yz+xz)

 Tentei,não consegui,peço ajuda.Desde já agradeço.

 --
 Esta mensagem foi verificada pelo sistema de antivírus e
 acredita-se estar livre de perigo.


-- 
Esta mensagem foi verificada pelo sistema de antivírus e
 acredita-se estar livre de perigo.



Re: [obm-l] Alguem sabe como resolver?

2014-02-20 Por tôpico saulo nilson
a formula esta errada nem tem soluçao para c=0 e cosH-1


2014-02-20 9:25 GMT-03:00 Rivaldo Dantas rbdantas...@yahoo.com.br:

 A substituição do valor na equação implica em obter uma nova equação de
 grau bem maior que a equação proposta, portanto não resolve o problema.
 Continua em aberto.

 Abs. Rivaldo.


   Em Quarta-feira, 19 de Fevereiro de 2014 18:37, carwatbr 
 carwa...@yahoo.com.br escreveu:

 Olá,  para esse problema, já tentou substituir o valor na equação?
 Saudações,
 Carlos Juiti Watanabe



  Mensagem original 
 De : douglas.olive...@grupoolimpo.com.br
 Data:18/02/2014 22:56 (GMT-03:00)
 Para: obm-l@mat.puc-rio.br
 Assunto: Re: [obm-l] Alguem sabe como resolver?

 Usa a fórmula de Cardano!!
 Lembro que já vi duas vezes nessa lista. a prova dela.

 Em 18.02.2014 22:10, Rivaldo Dantas escreveu:


  Fev 17 em 4:53 PM
Suponha que a equação  x^3+cx+d=0   admita apenas raízes racionais,
 onde c e d são números reais.
 Mostre que uma das raízes dessa equação é dada por

 x=(-3d/(2c))  -  (M)sqrt(-L)/(6c)  onde

 L=12c^3+81d^2  M= sen(p)/(1-cos(p))p= (1/3)arccos(H) e
 H=  (54d^2+4c^3)/(-4c^3)


 suponha também para evitar casos triviais que  o produto  cp  é diferente
 de zero.


 Rivaldo.

 Abs.

 --
 Esta mensagem foi verificada pelo sistema de antivírus e
 acredita-se estar livre de perigo.




 --
 Esta mensagem foi verificada pelo sistema de antiv�rus e
 acredita-se estar livre de perigo.
 --
 Esta mensagem foi verificada pelo sistema de antiv�us e
 acredita-se estar livre de perigo.



 --
 Esta mensagem foi verificada pelo sistema de antivírus e
 acredita-se estar livre de perigo.


-- 
Esta mensagem foi verificada pelo sistema de antiv�rus e
 acredita-se estar livre de perigo.



[obm-l] Re: [obm-l] RE: [obm-l] Re: [obm-l] Três de inteiros

2014-02-20 Por tôpico saulo nilson
foi.


2014-02-20 18:46 GMT-03:00 marcone augusto araújo borges 
marconeborge...@hotmail.com:

 Na segunda (m+n)^3 = m^3 + n^3 + 3mn(m+n)
 Foi isso que vc viu?

 --
 Date: Thu, 20 Feb 2014 13:47:48 -0300
 Subject: [obm-l] Re: [obm-l] Três de inteiros
 From: saulo.nil...@gmail.com
 To: obm-l@mat.puc-rio.br


 1--
 x^3+y^3=(x+y)(x^2-xy+y^2)=(x+y)^2
 x=-y
 ou
 x^2-xy+y^2-x-y=0
 delta=(1+y)^2-4y^2+4y=1+2y+y^2-4y^2+4y=1+6y-3y^2
 x=(1+y+-sqrt(4-3(y-1)^2))/-6nao serve pois nao tem soluçoes inteiras
 2--
 m+n=33
 3m^2n+3mn^2=99mn





 2014-02-20 11:23 GMT-03:00 marcone augusto araújo borges 
 marconeborge...@hotmail.com:

 1) Ache todos os pares de inteiros (x,y) tais que x^3 + y^3 = (x + y)^2

 2) Determine todos os pares de inteiros (m,n) tais que m.n  = 0 e
 m^3 + n^3 + 99mn = 33^3

 3) Se x,y,z são números reais não nulos,com x+y+z também não nulo
 Calcule os valores possíveis da expressão (x^2 + y^2 + z^2)/2(xy+yz+xz)

 Tentei,não consegui,peço ajuda.Desde já agradeço.

 --
 Esta mensagem foi verificada pelo sistema de antivírus e
 acredita-se estar livre de perigo.



 --
 Esta mensagem foi verificada pelo sistema de antivírus e
 acredita-se estar livre de perigo.

 --
 Esta mensagem foi verificada pelo sistema de antivírus e
 acredita-se estar livre de perigo.


-- 
Esta mensagem foi verificada pelo sistema de antivírus e
 acredita-se estar livre de perigo.



Re: [obm-l] Primos

2014-02-19 Por tôpico saulo nilson
(p+1)/2=Y^2
(p^2+1)/2=x^2
x^2-y^2=(x-y)(x+y)=p(p-1)/2
ab=(p-1)/2
x+y=ap
x-y=(p-1)/2a
x=(2a^2p+p-1)/4a=(p^2+1)/2
p=((2a^2+1)+-sqrt(4a^4-12a^2+1-8a)/4a
y=(2a^2p-p+1)/4a=(p+1)/2
p=(2a-1)/(2a^2-2a-1)
2a(2a-1)^2/(2a^2-2a-1)^2 -(2a^2+1)(2a-1)/(2a^2-2a-1)+2a+1==0
2a(2a-1)^2 -(2a^2+1)(2a-1)(2a^2-2a-1)+(2a+1)(2a^2-2a-1)^2=0
a=-1
a=0
a=2
p=1
 ou
x-y=(p-1)/2
x+y=p
x=(3p-1)/4
y=(p+1)/4
p=7


2014-02-18 23:18 GMT-03:00 marcone augusto araújo borges 
marconeborge...@hotmail.com:

 Certo...Pell.Tentei o seguinte:
 2m^2 =  n^2 + 1 *
 n é impar,então n = 2q + 1
 Substituindo n por 2p + 1 em* e arrumando:
 m^2 = q^2 + (q+1)^2(uma terna pitagorica)
 A unica terna pitagorica que conheço com os dois
 menores elementos sendo numeros consecutivos é (3,4,5)
 Dai q = 3,n = 7 e m = 5
 Não consegui´´usar que p é primo´´ nem saberia mostrar
 que os tais consecutivos só poderiam mesmo ser 3 e 4.



  Date: Tue, 18 Feb 2014 16:45:16 -0300
  Subject: Re: [obm-l] Primos
  From: bernardo...@gmail.com
  To: obm-l@mat.puc-rio.br

 
  2014-02-18 8:48 GMT-03:00 marcone augusto araújo borges
  marconeborge...@hotmail.com:
   Determine os primos p tais que (p+1)/2 e (p^2 + 1)/2 são quadrados
 perfeitos
  
   p = 2k + 1 = (p+1)/2 = k+1
   k+1 = t^2 = k = t^2 - 1 = p = 2t^2 - 1
   (p^2 +1)/2 = 2t^4 - 2t^2 + 1 = m^2
   2t^4 - 2t^2 + 1 - m^2 = 0
   Delta = 4(2m^2 - 1) = 2m^2 - 1 = n^2
   Deu pra ver que m = 5(e n = 7) satisfaz
   Dai t = 2,k = 3 e p = 7(há outro valor para p?)
   Como resolver mesmo 2m^2 - 1 = n^2 ?
 
  Isso dá uma equação de Pell. Mas acho que você talvez tenha que usar
  que p é primo em algum lugar, talvez seja mais simples do que resolver
  Pell.
 
  Abraços,
  --
  Bernardo Freitas Paulo da Costa
 
  --
  Esta mensagem foi verificada pelo sistema de antivírus e
  acredita-se estar livre de perigo.
 
 
  =
  Instruções para entrar na lista, sair da lista e usar a lista em
  http://www.mat.puc-rio.br/~obmlistas/obm-l.html
  =

 --
 Esta mensagem foi verificada pelo sistema de antivírus e
 acredita-se estar livre de perigo.


-- 
Esta mensagem foi verificada pelo sistema de antivírus e
 acredita-se estar livre de perigo.



[obm-l] Re: [obm-l] Congruências

2014-02-04 Por tôpico saulo nilson
300^1=300MOD1001
300^2=911MOD1001
300^3=27MOD1001
 =92MOD1001
=573MOD1001
==729MOD1001
482MOD1001
456MOD1001
664
1MOD1001
COMO 3000 E MULTIPLO DE 10
ENTAO
300^3000=1MOD1001


2014-02-04 marcone augusto araújo borges marconeborge...@hotmail.com:

 Determinar o resto da divisão de 300^3000 por 1001


 Pelos meus cálculos essa potência dividida por 7,por 11
 ou por 13 deixa o mesmo resto 1
 como 7,11 e 13 são primos e 7.11.13 = 1001,posso afirmar
 300^3000 dividido por 1001 deixa resto 1?

 --
 Esta mensagem foi verificada pelo sistema de antivírus e
 acredita-se estar livre de perigo.


-- 
Esta mensagem foi verificada pelo sistema de antivírus e
 acredita-se estar livre de perigo.



[obm-l] Re: [obm-l] Re: [obm-l] Re: [obm-l] derivação

2014-01-22 Por tôpico saulo nilson
y=cosx^x
lny=lncosx
y´/y=lncosx-xtgx
y´=cosx^x(lncosx-xtgx)


2014/1/22 Fabio Silva cacar...@yahoo.com

 Obrigado.

 Estava considerando como se fosse constante...mas é uma função tb.

 Valeu Bruno!


   On Tuesday, January 21, 2014 11:53 PM, Bruno França dos Reis 
 bfr...@gmail.com wrote:
  Para esse tipo de questão, o Wolfram Alpha é uma ferramenta excelente!
 Confira:
 http://www.wolframalpha.com/input/?i=derivative+of+%28cos%28x%29%29%5Ex

 Abs
 Bruno

 --
 Bruno FRANÇA DOS REIS

 tel: +55 11 9-9961-7732
 skype: brunoreis666

 http://brunoreis.com
 http://brunoreis.com/tech

 e^(pi*i)+1=0


 2014/1/21 Fabio Silva cacar...@yahoo.com

 Poderiam me dar a resposta correta, estou em dúvida:

 a derivada de (cos x)^x é:

 apenas (cos x)^x . ln (cos x)

 ou

 -sen x . (cos x)^x . ln (cos x)

 Obrigado

 Fabio MS

 --
 Esta mensagem foi verificada pelo sistema de antivírus e
 acredita-se estar livre de perigo.



 --
 Esta mensagem foi verificada pelo sistema de antiv�us e
 acredita-se estar livre de perigo.



 --
 Esta mensagem foi verificada pelo sistema de antivírus e
 acredita-se estar livre de perigo.


-- 
Esta mensagem foi verificada pelo sistema de antiv�rus e
 acredita-se estar livre de perigo.



Re: [obm-l] Inteiros(de novo)

2014-01-16 Por tôpico saulo nilson
 x^2 + y^2 = z^3 e x^2 + 4 = y^3
y^3+y^2-4=z^3
(-2,-2), (2,2)


2014/1/15 marcone augusto araújo borges marconeborge...@hotmail.com

 Onde encontro soluções de x^2 + y^2 = z^3 e x^2 + 4 = y^3?

 --
 Esta mensagem foi verificada pelo sistema de antivírus e
 acredita-se estar livre de perigo.


-- 
Esta mensagem foi verificada pelo sistema de antivírus e
 acredita-se estar livre de perigo.



Re: [obm-l] Inteiros(de novo)

2014-01-16 Por tôpico saulo nilson
(2,2,2)


2014/1/15 marcone augusto araújo borges marconeborge...@hotmail.com

 Onde encontro soluções de x^2 + y^2 = z^3 e x^2 + 4 = y^3?

 --
 Esta mensagem foi verificada pelo sistema de antivírus e
 acredita-se estar livre de perigo.


-- 
Esta mensagem foi verificada pelo sistema de antivírus e
 acredita-se estar livre de perigo.



Re: [obm-l] Quadrado perfeito

2014-01-15 Por tôpico saulo nilson
X^4+Y^4=Z^2
(x^2/z)^2+(y^2/z)^2=1
x^2/z=senp
y^2/z=cosp
comoo senp  e cosp sao numeros da foma  a/b ou sqrta/b ou (c+sqrta)/b com
-1=sena,cosa=1, com as 2 ultimas formas impossiveis de se encontrar x e z
inteiros, temos:
x^2/z=a/b , com a e b irredutiveis
pikn==produtorio de kn
x^2=za/b
para x ser inteiro
z=mb
x^2=am=a1a2a3...an*m1m2m3m...mm
tem que haver uma combinaçao entre an e mm de tal forma que se possa tirar
a raiz quadrada
(y^2/z)^2=1-  pimm^2pian^2/z^2
y^4=z^2-pimm^2pian^2=z^2-c^2=(z-c)*(z+c)=(mb-c)(mb+c)=m^2(b-a)(b+a)
b-a=mpikn
b+a=mpik´m
(b-a)/(b+a)=pikn/pikm
como kn e km se combinam para formar uma numero da forma k^4, uma das
maneiras de acontecer isto e:
(b-a)/(b+a)=1/d  ou c/d  dc
b(d-1)=a(d+1)
a/b=(d-1)/(d+1)
x^2/z=(d-1)/(d+1)
d=km -km intercessao kn, onde kn*km=k^4
x^2=m^2(pikn +pikm)(d-1)/2(d+1)=m^2pikn(d-1)=m^2(pikm
-pikn)=m^2(pikn^3pikm´^4-pikn)
o caso mais facil e quando pikm=k^3
x^2=m^2k(k^2-1)=m^2(k-1)k(k+1)
impossivel de encontrar um quadrado perfeito pois entre 3 numeros
consecutivos sempre sobram numeros primos com expoentes diferentes de 2,
restando numeros irracionais.
no caso mais dificil
x^2=m^2k(k^2w^4-1)=m^2(knw^2-1)kn(knw^2+1) , como entre 2 numoros quase
consecutivos , a-1, a+1, quando fatorados sempre temos no minimo 2 primos
diferentes, e kn e menor do que knw^2, fatorando kn encontraremos primos
menores do que a fatoração de kn2^2+1 e knw^2-1, o que nos resta no minimo
3 primos com expoentes diferentes de 2 ou multiplos de 2, restando um
numeros irracional da forma
x=x´*sqrt(xp1 xp2 xp3)





2014/1/15 marcone augusto araújo borges marconeborge...@hotmail.com

 Obrigado!

 --
 Date: Wed, 15 Jan 2014 07:05:58 -0800
 From: luizfelipec...@yahoo.com.br

 Subject: Re: [obm-l] Quadrado perfeito
 To: obm-l@mat.puc-rio.br


 Sugestão :

 Use as soluções gerais :

 z = a^2+b^2
 y2 = a^2-b^2
 x^2= 2ab

 Verifique agoa, se vc consegue aplicar o metodo da descida infinita.

 Abs
 Felipe



   Em Quarta-feira, 15 de Janeiro de 2014 12:32, marcone augusto araújo
 borges marconeborge...@hotmail.com escreveu:
  Esqueçam o que falei sobre a soma de 2 quartas potências,tá errado.
 continuo sem conseguir a solução.

 --
 From: marconeborge...@hotmail.com
 To: obm-l@mat.puc-rio.br
 Subject: RE: [obm-l] Quadrado perfeito
 Date: Wed, 15 Jan 2014 12:48:24 +

 Eu notei depois que agente pode mostrar que a soma de duas quartas
 potências
 está entre dois quadrados consecutivos,portanto não pode ser um quadrado
 Tentei por  congruência mas por esse caminho não saiu
 Não entendi seu raciocínio,Saulo.


 --
 Date: Wed, 15 Jan 2014 02:27:37 -0200
 Subject: Re: [obm-l] Quadrado perfeito
 From: saulo.nil...@gmail.com
 To: obm-l@mat.puc-rio.br

 x^4+y^4=z^2
 x^2+y^2z
 y^2+zx^2
 x^2+z^y^2
 dai nos encontramos
 x^2z
 y^2z
 onde se conclui que a igualdades e uma contradiçao, pois x^4+z^4z^2


 2014/1/14 marcone augusto araújo borges marconeborge...@hotmail.com

 Mostre que a equação X^4 + Y^4 = Z^2 não tem solução nos inteiros positivos
 Tô tentando sem sucesso.

 --
 Esta mensagem foi verificada pelo sistema de antivírus e
 acredita-se estar livre de perigo.



 --
 Esta mensagem foi verificada pelo sistema de antivírus e
 acredita-se estar livre de perigo.

 --
 Esta mensagem foi verificada pelo sistema de antivírus e
 acredita-se estar livre de perigo.

 --
 Esta mensagem foi verificada pelo sistema de antivírus e
 acredita-se estar livre de perigo.



 --
 Esta mensagem foi verificada pelo sistema de antivírus e
 acredita-se estar livre de perigo.

 --
 Esta mensagem foi verificada pelo sistema de antivírus e
 acredita-se estar livre de perigo.


-- 
Esta mensagem foi verificada pelo sistema de antivírus e
 acredita-se estar livre de perigo.



Re: [obm-l] Quadrado perfeito

2014-01-14 Por tôpico saulo nilson
x^4+y^4=z^2
x^2+y^2z
y^2+zx^2
x^2+z^y^2
dai nos encontramos
x^2z
y^2z
onde se conclui que a igualdades e uma contradiçao, pois x^4+z^4z^2


2014/1/14 marcone augusto araújo borges marconeborge...@hotmail.com

 Mostre que a equação X^4 + Y^4 = Z^2 não tem solução nos inteiros positivos
 Tô tentando sem sucesso.

 --
 Esta mensagem foi verificada pelo sistema de antivírus e
 acredita-se estar livre de perigo.


-- 
Esta mensagem foi verificada pelo sistema de antivírus e
 acredita-se estar livre de perigo.



Re: [obm-l] forma geral da conica

2014-01-13 Por tôpico saulo nilson
tem que fazer uma rotação de eixos para ficar na forma de conica normal.


2014/1/13 Luís qed_te...@hotmail.com

 Sauda,c~oes,

 Seja a cônica dada pela equação

 Ax^2 + Bxy + Cy^2 + Dx + Ey + F = 0 (B/=0)

 Como expressar os parâmetros da cônica (foco, centro, diretriz etc)
 em função dos parâmetros da equação ?

 Estou perguntando simbolicamente pois numericamente eu tenho
 estas informações com o Wolfram Alpha, por exemplo.

 Fiz um teste com 5x^2 + 2xy - 5y^2 + 6x + 8y + 12 = 0

 e soh não vi claramente apresentada a diretriz.

 Luis


 --
 Esta mensagem foi verificada pelo sistema de antivírus e
 acredita-se estar livre de perigo.


-- 
Esta mensagem foi verificada pelo sistema de antivírus e
 acredita-se estar livre de perigo.



[obm-l] Re: [obm-l] Composição de Funções periódicas

2014-01-13 Por tôpico saulo nilson
se sen x e periodica entao
sen0=sen2pi=sen4pi=sen6i periodo 2pi
se pegarmos
x=sqrt deses4s valores
entao temos
sen(0),sensqrt2pi^2,sensqrt4pi^2,sensqrt6pi^2 que e periodica tambem


2014/1/13 Artur Costa Steiner steinerar...@gmail.com

 gof é periódica. Se t é período de f, então, para todo x, gof(x + t) =
 g(f(x + t)) = g(f(x) = gof(x), de modo que t é período de gof.

 fog não tem que ser periódica. Por exemplo, se f(x) = senx e g(x) = x^2,
 então f é periódica mas fog(x) = sen(x^2) não é.

 Artur Costa Steiner

  Em 10/01/2014, às 08:17, Gabriel Ayres do Nascimento 
 gan_ay...@yahoo.com.br escreveu:
 
 
  Fala pessoal,
 
  Seja f uma função periódica de R em R e g uma função qualquer de R em R.
 A função composta gof é necessariamente periódica? E a função fog?
 Demonstre, caso afirmativo, ou dê um contra exemplo, cado contrário.
 
  Deem uma ideia aí.
 
  Gabriel Ayres
 
  --
  Esta mensagem foi verificada pelo sistema de antivírus e
  acredita-se estar livre de perigo.
 
 
  =
  Instruções para entrar na lista, sair da lista e usar a lista em
  http://www.mat.puc-rio.br/~obmlistas/obm-l.html
  =

 --
 Esta mensagem foi verificada pelo sistema de antivírus e
  acredita-se estar livre de perigo.


 =
 Instru�ões para entrar na lista, sair da lista e usar a lista em
 http://www.mat.puc-rio.br/~obmlistas/obm-l.html
 =


-- 
Esta mensagem foi verificada pelo sistema de antiv�rus e
 acredita-se estar livre de perigo.



[obm-l] Re: [obm-l] Composição de Funções periódicas

2014-01-12 Por tôpico saulo nilson
g(f(x))=g(f(x+T)) n ao necessariamente periodica
f(g(x))=f(g(x)+T1) periodica


2014/1/10 Gabriel Ayres do Nascimento gan_ay...@yahoo.com.br


 Fala pessoal,

 Seja f uma função periódica de R em R e g uma função qualquer de R em R. A
 função composta gof é necessariamente periódica? E a função fog? Demonstre,
 caso afirmativo, ou dê um contra exemplo, cado contrário.

 Deem uma ideia aí.

 Gabriel Ayres

 --
 Esta mensagem foi verificada pelo sistema de antivírus e
  acredita-se estar livre de perigo.


 =
 Instruções para entrar na lista, sair da lista e usar a lista em
 http://www.mat.puc-rio.br/~obmlistas/obm-l.html
 =


-- 
Esta mensagem foi verificada pelo sistema de antivírus e
 acredita-se estar livre de perigo.



[obm-l] Re: [obm-l] Composição de Funções periódicas

2014-01-12 Por tôpico saulo nilson
g(f(x))=g(f(x+T))=g(f(x)+T1)
T1=f(x+T)-f(x) pode ser ou nao periodica



2014/1/12 saulo nilson saulo.nil...@gmail.com

 g(f(x))=g(f(x+T)) n ao necessariamente periodica
 f(g(x))=f(g(x)+T1) periodica


 2014/1/10 Gabriel Ayres do Nascimento gan_ay...@yahoo.com.br


 Fala pessoal,

 Seja f uma função periódica de R em R e g uma função qualquer de R em R.
 A função composta gof é necessariamente periódica? E a função fog?
 Demonstre, caso afirmativo, ou dê um contra exemplo, cado contrário.

 Deem uma ideia aí.

 Gabriel Ayres

 --
 Esta mensagem foi verificada pelo sistema de antivírus e
  acredita-se estar livre de perigo.


 =
 Instruções para entrar na lista, sair da lista e usar a lista em
 http://www.mat.puc-rio.br/~obmlistas/obm-l.html
 =




-- 
Esta mensagem foi verificada pelo sistema de antivírus e
 acredita-se estar livre de perigo.



[obm-l] Re: [obm-l] Composição de Funções periódicas

2014-01-12 Por tôpico saulo nilson
f(g(x))=f(g(x)+T)  periodica
g(f(x))=g(f(x+T))  periodica


2014/1/10 Gabriel Ayres do Nascimento gan_ay...@yahoo.com.br


 Fala pessoal,

 Seja f uma função periódica de R em R e g uma função qualquer de R em R. A
 função composta gof é necessariamente periódica? E a função fog? Demonstre,
 caso afirmativo, ou dê um contra exemplo, cado contrário.

 Deem uma ideia aí.

 Gabriel Ayres

 --
 Esta mensagem foi verificada pelo sistema de antivírus e
  acredita-se estar livre de perigo.


 =
 Instruções para entrar na lista, sair da lista e usar a lista em
 http://www.mat.puc-rio.br/~obmlistas/obm-l.html
 =


-- 
Esta mensagem foi verificada pelo sistema de antivírus e
 acredita-se estar livre de perigo.



Re: [obm-l] Inteiros

2014-01-11 Por tôpico saulo nilson
3^p^2+3^h^2+1=t^2
3^h^2+1 deve ser um numero quadratico senao nao existe um triangulo com 3^m
, 3^n+1 e t
3^h^2=k^2-1=(k-1)(k+1) que e impossivel pois os numeros da forma 3^m nao
podem ser colocados como produtos de numeros quase consecutivos.


2014/1/8 marcone augusto araújo borges marconeborge...@hotmail.com

 Mostre que a equação 3^m + 3^n + 1 = t^2 não tem solução nos inteiros


 a = b (c) significa a é congruo a b modulo c

 o primeiro membro da equação representa um numero impar,então t é impar
 chamando t de 2k+1 e desenvolvendo temos 3^m + 3^n = 4k(k+1) *
 o segundo membro de * é um multiplo de 8
 o primeiro membro de * nunca é multiplo de 8,pois de 3^2 = 1(8),segue que
 3^(2p) = 1(8) e 3^(2p + 1) = 3(8)
 Dai :  a) 3^m + 3^n = 6(8) se m e n são impares
b) 3^m + 3^n = 2(8) se m e n são pares
c) 3^m + 3^n = 4(8) se um dos expoentes é par e o outro,impar
 Eu agradeceria  se alguem apresentasse uma solução diferente.



 --
 Esta mensagem foi verificada pelo sistema de antivírus e
 acredita-se estar livre de perigo.


-- 
Esta mensagem foi verificada pelo sistema de antivírus e
 acredita-se estar livre de perigo.



Re: [obm-l] Como eu resolvo isso?

2014-01-04 Por tôpico saulo nilson
d2R/R=2d2acosa/sena
lnR dR=2(lnsena+V/D)da
RlnR-R+D=2aV/D+2Integral (lnsenada)


2014/1/3 João Maldonado joao_maldona...@hotmail.com

 Fala ai galera. Eu tava resolvendo um problema de cinemática (sei que não
 é o assunto da lista) mas caí numa parte puramente matemática que não estou
 conseguindo resolver, queria pedir a ajuda de vocês. Se alguém puder me dar
 uma mão eu agradeço muito

 d²R/dt² = 2cos(a)/R³
 R d²a/dt² = sen(a)/R³

 onde: R inicial = D
 a inicial = 0
 dR/dt inicial = 0
 da/dt inicial = V/D

 Quero achar R(t), a(t) e R(a)

 []'s
 João

 --
 Esta mensagem foi verificada pelo sistema de antivírus e
 acredita-se estar livre de perigo.


-- 
Esta mensagem foi verificada pelo sistema de antivírus e
 acredita-se estar livre de perigo.



Re: [obm-l] Como eu resolvo isso?

2014-01-04 Por tôpico saulo nilson
essa integral e catalogada caiu em uma prova da obmu.


2014/1/4 saulo nilson saulo.nil...@gmail.com

 d2R/R=2d2acosa/sena
 lnR dR=2(lnsena+V/D)da
 RlnR-R+D=2aV/D+2Integral (lnsenada)


 2014/1/3 João Maldonado joao_maldona...@hotmail.com

 Fala ai galera. Eu tava resolvendo um problema de cinemática (sei que não
 é o assunto da lista) mas caí numa parte puramente matemática que não estou
 conseguindo resolver, queria pedir a ajuda de vocês. Se alguém puder me dar
 uma mão eu agradeço muito

 d²R/dt² = 2cos(a)/R³
 R d²a/dt² = sen(a)/R³

 onde: R inicial = D
 a inicial = 0
 dR/dt inicial = 0
 da/dt inicial = V/D

 Quero achar R(t), a(t) e R(a)

 []'s
 João

 --
 Esta mensagem foi verificada pelo sistema de antivírus e
 acredita-se estar livre de perigo.




-- 
Esta mensagem foi verificada pelo sistema de antivírus e
 acredita-se estar livre de perigo.



[obm-l] Re: [obm-l] Re: [obm-l] Re: [obm-l] Cálculo

2013-12-29 Por tôpico saulo nilson
*Sejam f e g funções contínuas num intervalo [a, b], tais que f(a)  g(a) e
f(b)  g(b). Prove que existe um número c entre a e b, tal que f(c) = g(c).*
*f(a)=g(a)-h*
*f(b)=g(b)+h*
*se f  e funçao e e continua entao o teorema tem que ser valido para
f(x)=c´x+d,g(x)=ex+f*
*f(a)=c´a+d*
*f(b)=c´b+d*
*c´=(f(b)-f(a))/(b-a)*
*da mesma forma*
*e=(g(b)-g(a))/(a-b)*
*como c´=e+2h/(b-a) nao sao paralelas elas tem uma intercessao entre a e b
tal que f(c)=g(c)*


2013/12/25 Vanderlei Nemitz vanderma...@gmail.com

 Se h(a)  0 e h(b)  0, então pelo TVI, existe um c tal que h(c) = 0?
 Correto esse raciocínio?


 Em 25 de dezembro de 2013 15:29, Gabriel Haeser ghae...@gmail.comescreveu:

 Defina h=f-g e use o teorema do valor intermediario.


 On Wednesday, December 25, 2013, Vanderlei Nemitz wrote:

 Alguém poderia me ajudar na seguinte questão? Muito obrigado e um feliz
 Natal!

 *Sejam f e g funções contínuas num intervalo [a, b], tais que f(a) 
 g(a) e f(b)  g(b). Prove que existe um número c entre a e b, tal que f(c)
 = g(c).*

 --
 Esta mensagem foi verificada pelo sistema de antivírus e
 acredita-se estar livre de perigo.


 --
 Esta mensagem foi verificada pelo sistema de antivírus e
 acredita-se estar livre de perigo.



 --
 Esta mensagem foi verificada pelo sistema de antivírus e
 acredita-se estar livre de perigo.


-- 
Esta mensagem foi verificada pelo sistema de antivírus e
 acredita-se estar livre de perigo.



Re: [obm-l] x+ r tende a mais infinito

2013-12-29 Por tôpico saulo nilson
lim f   =L
x--oo

|x-e|D
|f(x)|L
|x+r-e||x-e|+|r|=||x-e|D
lim f===  lim f
x--oo x+r==00



2013/12/29 Ennius Lima enn...@bol.com.br

 Caros Colegas,

 Como podemos provar que são equivalentes as afirmações x tende a mais
 infinito e x + r tende a mais infinito?   ( x é uma variável real, r é
 uma constante real.)

 Feliz Anovo Novo para todos!
 Ennius Lima
 


 --
 Esta mensagem foi verificada pelo sistema de antivírus e
  acredita-se estar livre de perigo.

 =
 Instru�ões para entrar na lista, sair da lista e usar a lista em
 http://www.mat.puc-rio.br/~obmlistas/obm-l.html
 =


-- 
Esta mensagem foi verificada pelo sistema de antiv�rus e
 acredita-se estar livre de perigo.



  1   2   3   4   5   6   7   >